You are on page 1of 57

TEST -4 (3181)

VISION IAS
PT TEST -4
(3181)
Geography – II

2021
SYLLABUS

Economic and Human Geography - World


▪ Economic Geography of the world: primary, secondary, tertiary,
quaternary and quinary activities
▪ Human Geography of world
▪ World Population, Distribution & Density, Races & Tribes,
Settlement & Migration

Economic and Human Geography - India


▪ Agriculture
▪ Land Resources
▪ Water Resources
▪ Mineral and Energy Resources
▪ Industries
▪ Transport and Communication
▪ Foreign Trade
▪ Population, Migration, Settlements

+ Current Affairs (April-May 2020)

REFERENCES

▪ 12th NCERT: Geography - India People and Economy


▪ 12th NCERT: Fundamental of Human Geography
▪ 10th NCERT: Contemporary India- Part-II
▪ Certificate Physical and Human Geography - Goh Cheng Leong
▪ Orient Longman Atlas or Oxford Atlas
▪ Vision IAS Value Addition Material Eco. Geography
TEST 4
https://t.me/UPSC_PDF Download From > https://upscpdf.com https://t.me/UPSC_PDF

1. Which of the following conditions can be 5. Prana-Vayu seen recently in news is related
considered favourable for the development to:
of market gardening in the British Type of (a) An indigenously developed air pollution
climate? monitoring device.
1. Highly industrialized and high
(b) N95 variant masks with a double layer
population densities
protection.
2. A good network of transport
(c) A low-cost portable ventilator
Select the correct answer using the code
given below. (d) A device to measure the HCFC level in
(a) 1 only summers emitted due to air conditioners.
(b) 2 only
(c) Both 1 and 2 6. Which of the following activities are called
(d) Neither 1 nor 2 gold-collar professions?
(a) Quinary activities
2. Which of the following families of modern (b) Quaternary activities
Indian languages has the highest number of (c) Secondary activities
speakers in India?
(d) Primary activities
(a) Austric
(b) Dravidian
7. With reference to the growth trends of the
(c) Sino-Tibetan
(d) Indo-European global population, consider the following
statements:
3. Consider the following statements: 1. The doubling time of world population
1. Sugar Industry is a seasonal industry. has been increasing in the last century.
2. India is the largest producer of 2. At the beginning of the twentieth
sugarcane in the world. century, the world population was less
3. The crop yield of sugarcane is higher in than half of its current levels.
Northern India than in Southern India. Which of the statements given above is/are
Which of the statements given above is/are
correct?
correct?
(a) 1 only
(a) 1 only
(b) 2 only
(b) 2 and 3 only
(c) 1 and 3 only (c) Both 1 and 2
(d) 1, 2 and 3 (d) Neither 1 nor 2

4. Which of the statements given below is 8. The term EventBot recently seen in news is:
correct regarding the cotton industry? (a) a malware steals personal financial
(a) Cotton is a non-weight losing raw information from Android phone users
material. (b) a drone that is capable of bringing down
(b) Cotton industries are mostly market-
swarms of locusts by spraying pesticide
based industries.
(c) a software to aid government to collect
(c) Maharashtra, Gujarat and Andhra
data of all public sector units.
Pradesh are leading producers of cotton.
(d) an event organised by India to showcase
(d) All the statements (a), (b) and (c) are
correct. their robotics technology.

2 www.visionias.in ©Vision IAS

Google it:- https://upscpdf.com


https://t.me/UPSC_PDF Download From > https://upscpdf.com https://t.me/UPSC_PDF

9. Consider the following statements about 12. Which of the following are metallic
sedimentary basins in India: minerals?
1. In India they are located only along with 1. Copper
the offshore areas. 2. Graphite
2. They are an important source of oil, 3. Bauxite
natural gas, and petroleum. 4. Limestone
Which of the statements given above is/are Select the correct answer using the code
not correct? given below.
(a) 1 only (a) 1 and 2 only

(b) 2 only (b) 1 and 3 only

(c) Both 1 and 2 (c) 3 only

(d) Neither 1 nor 2 (d) 1, 2 and 4 only

10. Consider the following statements: 13. Consider the following statements:

1. The maximum percentage of surface 1. Industries related to atomic energy are

water in India is utilized by the industrial completely under the public sector.

sector. 2. No industry requires prior approval from


the Government for investing in the
2. The maximum percentage of
delicensed sector.
groundwater in India is utilized by the
Which of the statements given above is/are
agriculture sector.
correct?
Which of the statements given above is/are
(a) 1 only
correct?
(b) 2 only
(a) 1 only
(c) Both 1 and 2
(b) 2 only
(d) Neither 1 nor 2
(c) Both 1 and 2
(d) Neither 1 nor 2
14. Which of the following statements are
correct regarding the Direct Seeding of Rice
11. Which of the following factors play a role in
(DSR) technique?
determining the nature of rural settlements in
1. It is a process of establishing a rice crop
India?
from seeds sown in field rather than by
1. Nature of terrain
transplanting seedlings from nursery.
2. Caste and ethnicity of people
2. This technique reduces greenhouse gas
3. Availability of water
emissions.
Select the correct answer using the code
Select the correct answer using the code
given below.
given below.
(a) 1 and 2 only
(a) 1 only
(b) 2 and 3 only
(b) 2 only
(c) 1 and 3 only
(c) Both 1 and 2
(d) 1, 2 and 3 (d) Neither 1 nor 2
3 www.visionias.in ©Vision IAS

Google it:- https://upscpdf.com


https://t.me/UPSC_PDF Download From > https://upscpdf.com https://t.me/UPSC_PDF

15. Which of the following are identified shale 18. Consider the following :
gas basins for exploration? This stage in the demographic transition
1. Cambay theory is marked by high birth rate but
2. K-G Basin lowering death rates resulting in a natural

3. Assam-Arakan increase in the population. Countries like

4. Indus River Basin Kenya and Sri Lanka are currently said to be

Select the correct answer using the code in this stage.


Which of the following stages of the
given below.
demographic transition theory is being
(a) 1 and 4 only
described in the above passage?
(b) 1, 2 and 3 only
(a) Stage I
(c) 2, 3 and 4 only
(b) Stage II
(d) 1, 2, 3 and 4
(c) Stage III
(d) None
16. Geofencing, recently seen in the news, is
(a) the banning of mines of precious metals
19. Consider the following pairs:
for private sector extraction.
Settlement Region
(b) the encircling of the underground 1. Dispersed : Himachal Pradesh
facilities for nuclear waste disposal. 2. Clustered : Bundelkhand
(c) a location-based service in which an app 3. Hamleted : Lower Ganga Plains
or software uses GPS, RFID, Wi-Fi or Which of the pairs given above is/are
cellular data to trigger a pre- correctly matched?
programmed action. (a) 1 only
(d) a geographical area demarcated to track (b) 2 and 3 only
movement of wild animals in protected (c) 1, 2 and 3
areas. (d) None

17. Consider the following pairs: 20. Consider the following statements in the

Nuclear Plant State context of the World Health Assembly:

1. Tarapur : Maharashtra 1. It is attended by delegations from all the


WHO Member States.
2. Rawatbhata : Rajasthan
2. It is held in Geneva annually.
3. Kalpakkam : Karnataka
3. It implements the policies and decisions
4. Kaiga : Tamil Nadu
of WHO.
Which of the pairs given above is/are
Which of the statements given above is/are
correctly matched?
correct?
(a) 1, 2 and 3 only
(a) 1 only
(b) 1 and 4 only
(b) 1 and 2 only
(c) 1 and 2 only
(c) 2 and 3 only
(d) 2 and 3 only
(d) 3 only
4 www.visionias.in ©Vision IAS

Google it:- https://upscpdf.com


https://t.me/UPSC_PDF Download From > https://upscpdf.com https://t.me/UPSC_PDF

21. Which of the following statements best 24. Consider the following statements about the
describes the truck farming? Human Development Index:
(a) It is a farming in which farmers 1. It is issued every year by the World
specialise in vegetables. Bank.
2. It is calculated as a geometric mean of
(b) It is a farming in which animals are
Health, Education & Standard of Living
transported from one place to another for
Indicators.
trading.
Which of the statements given above is/are
(c) It is a farming in which heavy machinery
correct?
is used for harvesting the crops. (a) 1 only
(d) It is a part of dairy farming in which (b) 2 only
trucks are used for transporting milk (c) Both 1 and 2
from one place to another. (d) Neither 1 nor 2

22. Consider the following statements regarding 25. With respect to the agriculture in the
Iron and Steel Industries: Mediterranean region, consider the
1. All the raw materials used in the iron following statements:
1. The Mediterranean lands are known as
and steel industry are non-weight losing
the world’s orchard lands.
raw materials.
2. Viticulture is a traditional occupation of
2. In India, all steel plants are located near
the Mediterranean region.
the sites of raw material. 3. In mountainous regions transhumance is
Which of the statements given above is/are widely practiced.
correct? Which of the statements given above is/are
(a) 1 only correct?
(b) 2 only (a) 1 only
(c) Both 1 and 2 (b) 1 and 3 only
(d) Neither 1 nor 2 (c) 2 and 3 only
(d) 1, 2 and 3

23. Consider the following statements regarding


26. Consider the following statements about
sex ratio in labour force participation in
Watershed Management:
India:
1. It only includes the efficient
1. In India, females outnumber males in
management and conservation of surface
labour force participation in urban areas.
water only.
2. There are more number of females 2. It involves building up storage and
engaged in the primary sector in India recharge structures.
than males. 3. Community participation is an important
Which of the statements given above is/are component of Watershed Management.
correct? Which of the statements given above is/are
(a) 1 only correct?
(b) 2 only (a) 1 and 2 only
(b) 2 and 3 only
(c) Both 1 and 2
(c) 1 and 3 only
(d) Neither 1 nor 2
(d) 1, 2 and 3
5 www.visionias.in ©Vision IAS

Google it:- https://upscpdf.com


https://t.me/UPSC_PDF Download From > https://upscpdf.com https://t.me/UPSC_PDF

27. With reference to the Demographic 30. Consider the following statements :
Transition Theory, consider the following 1. Long and Sunny summers help in easy
statements: ripening of fruits.
1. According to the theory, the population 2. Fruit trees have long roots enabling them
of a given region changes from high to survive the dry season.
births and high deaths to low births and
3. Thick leathery bark of many fruit
low deaths.
trees prevents excessive transpiration.
2. During a complete demographic cycle,
Which of the statements given above is/are
rural agrarian societies are transformed
the favorable factors for Orchard farming in
into urbanized and industrial economies.
the Mediterranean region?
Which of the statements given above is/are
correct? (a) 1 only
(a) 1 only (b) 2 and 3 only
(b) 2 only (c) 1 and 3 only
(c) Both 1 and 2 (d) 1, 2 and 3
(d) Neither 1 nor 2
31. Consider the following pairs:
28. Arrange the following religions in increasing Mineral State Associated
order of the number of people practising 1. Mica : Jharkhand
them in India? 2. Copper : Rajasthan
1. Sikhs 3. Anthracite Coal : Tamil Nadu
2. Christians
Which of the pairs given above are correctly
3. Muslims
matched?
4. Buddhists
(a) 1 and 2 only
Select the correct answer using the code
(b) 2 and 3 only
given below.
(a) 2-1-3-4 (c) 1 and 3 only

(b) 3-1-2-4 (d) 1, 2 and 3


(c) 3-2-1-4
(d) 1-2-3-4 32. Consider the following statements regarding
the growth of India's population:
29. Consider the following statements about 1. The decadal growth rate of India's
Quinary Activities: population is constantly increasing since
1. The highest level of decision-makers or independence.
policymakers perform quinary activities. 2. India has never witnessed a decrease in
2. Banking and Legal Advisory are prime its decadal population in the last century.
examples of quinary activities.
Which of the statements given above is/are
Which of the statements given above is/are
correct?
correct?
(a) 1 only
(a) 1 only
(b) 2 only
(b) 2 only
(c) Both 1 and 2
(c) Both 1 and 2
(d) Neither 1 nor 2 (d) Neither 1 nor 2
6 www.visionias.in ©Vision IAS

Google it:- https://upscpdf.com


https://t.me/UPSC_PDF Download From > https://upscpdf.com https://t.me/UPSC_PDF

33. The air services between 10-35 degree South 36. Consider the following statements:
latitudes is very limited because: 1. The area under pulses cultivation is
1. Low density of population. greater than the area under the

2. High continentality, supporting other cultivation of oilseeds and coarse


cereals.
modes of land transport.
2. The area under wheat cultivation is the
3. Low economic development.
highest.
Which of the statements given above is/are
3. The area under jute cultivation is less
correct?
than the area under cotton cultivation.
(a) 1 and 2 only Which of the statements given above is/are
(b) 1 and 3 only correct?
(c) 3 only (a) 2 and 3 only
(d) 2 and 3 only (b) 3 only
(c) 1 and 2 only
34. Which of the following regions are major (d) 1, 2 and 3
industrial regions in India?
1. Mumbai-Pune Region 37. Which of the following countries are a part
of Snow Leopard Range Countries?
2. Vishakhapatnam-Guntur Region
1. Mongolia
3. Chhotanagpur Region
2. Russia
4. Ambala-Amritsar Region
3. Afghanistan
Select the correct answer using the code
4. India
given below.
Select the correct answer using the code
(a) 1, 2 and 3 only given below.
(b) 1 and 3 only (a) 2, 3 and 4 only
(c) 2 and 4 only (b) 1 and 2 only
(d) 1, 2, 3 and 4 (c) 1, 3 and 4 only
(d) 1, 2, 3 and 4
35. In the Himalayas, some tribes like Gujjars,
Bakarwals, Gaddis, and Bhotiyas migrate 38. Consider the following statements:

from plains to the mountains in summers and 1. Coal accounts for more than half of
India's energy needs.
to the plains from the high altitude pastures
2. Most of India's coal reserves are of non-
in winters in every season.
coking grade.
Which of the following practice is described
Which of the statements given above is/are
in the above statement?
correct?
(a) Jhuming (a) 1 only
(b) Transhumance (b) 2 only
(c) Gathering (c) Both 1 and 2
(d) Pastoralism (d) Neither 1 nor 2
7 www.visionias.in ©Vision IAS

Google it:- https://upscpdf.com


https://t.me/UPSC_PDF Download From > https://upscpdf.com https://t.me/UPSC_PDF

39. Consider the following pairs: 42. Which of the following conditions has led to
Crop/Tree Leading Producer Country the establishment of plantations in the
equatorial regions?
1. Palm oil : Indonesia
1. Favourable hot and wet climate.
2. Jute : Bangladesh 2. Nutrient-rich tropical soils.
3. Teak : India 3. Less problem of tree diseases.
Select the correct answer using the code
Which of the pairs given above is/are
given below.
correctly matched? (a) 1 only
(a) 1 only (b) 1, 2 and 3
(b) 2 and 3 only (c) 2 and 3 only
(d) 1 and 3 only
(c) 1 and 2 only
(d) 1, 2 and 3
43. Consider the following statements with
regard to styrene, a flammable substance,
40. Consider the following statements: recently in the news:
1. Migration takes place only when people 1. It is a flammable substance that is used
in the manufacturing of polystyrene
move from rural to urban areas in search
plastics, fibreglass, rubber, and latex.
of employment. 2. It is found in vehicle exhaust, cigarette
2. In India, intra-state rural to rural smoke, and in natural foods like fruits
and vegetables.
migration by place of last residence is
3. It is a highly toxic substance that can
dominated by females. lead to respiratory system collapse.
Which of the statements given above is/are Which of the statements given above is/are
correct? correct?
(a) 1 only
(a) 1 only
(b) 2 and 3 only
(b) 2 only (c) 1 and 2 only
(c) Both 1 and 2 (d) 1, 2 and 3
(d) Neither 1 nor 2
44. Consider the following statements in the
context of National Rural Infrastructure
41. Consider the following activities:
Development Agency, an agency associated
1. Hunting and gathering with many government programmes:
2. Pastoral activities 1. It is an agency under the administrative
control of the Ministry of Rural
3. Fishing
Development.
4. Forestry 2. It has recently announced the utilisation
5. Mining and quarrying of coir geotextiles for the construction of
Which of the activities mentioned above are rural roads under the Pradhan Mantri
Gram Sadak Yojana (PMGSY-III).
considered primary activities?
Which of the statements given above is/are
(a) 3 and 4 only correct?
(b) 1, 2 and 3 only (a) 1 only
(b) 2 only
(c) 2, 4 and 5 only
(c) Both 1 and 2
(d) 1, 2, 3, 4 and 5 (d) Neither 1 nor 2
8 www.visionias.in ©Vision IAS

Google it:- https://upscpdf.com


https://t.me/UPSC_PDF Download From > https://upscpdf.com https://t.me/UPSC_PDF

Google it:- https://upscpdf.com


https://t.me/UPSC_PDF Download From > https://upscpdf.com https://t.me/UPSC_PDF

Google it:- https://upscpdf.com


https://t.me/UPSC_PDF Download From > https://upscpdf.com https://t.me/UPSC_PDF

57. Consider the following statements about 60. Consider the following pairs:
types of crude oil: Cropping Season Crop
1. Sour crude contains a high amount of 1. Kharif Season : Wheat
sulfur content while sweet crude 2. Rabi Season : Cotton
contains relatively less amount of sulfur 3. Zaid Season : Cucumber
content. Which of the pairs given above is/are
2. Iraq is one of the largest producers of correctly matched?
sweet crude oil. (a) 1 and 2 only
Which of the statements given above is/are (b) 2 and 3 only
correct? (c) 3 only
(a) 1 only (d) 1, 2 and 3
(b) 2 only
(c) Both 1 and 2 61. Lumbering is defined as the felling of
(d) Neither 1 nor 2 economic trees in the forest, which can be
used for domestic, industrial, or commercial
58. Which of the following are the purposes. In this context, consider the
characteristics of the footloose industry? following statements:
1. They are dependent on a specific raw 1. Summer is the favorable season for
material. lumbering activity.
2. They are generally non-polluting 2. The heaviness of tropical timber is a
industries. hindering factor for lumbering in tropics.
Select the correct answer using the code 3. The Siberian region has a thriving
given below. lumbering industry.
(a) 1 only Which of the statements given above is/are
(b) 2 only correct?
(c) Both 1 and 2 (a) 1 and 3 only
(d) Neither 1 nor 2 (b) 1, 2 and 3
(c) 3 only
59. Which of the following is/are 'Push Factors' (d) 2 and 3 only
responsible for the mass exodus of the
migrant laborers from urban areas? 62. Consider the following pairs:
1. Better living conditions in rural areas. Tribes Regions
2. Epidemics in urban areas. 1. Pygmies : Malaysia
3. Lack of employment opportunities in 2. Indian : Amazon Basin
rural areas. 3. Orang Asli : Congo Basin
Select the correct answer using the code Which of the pairs given above is/are
given below. correctly matched?
(a) 1 and 2 only (a) 1 only
(b) 2 only (b) 1 and 2 only
(c) 1 and 3 only (c) 2 and 3 only
(d) 1, 2 and 3 (d) 2 only
11 www.visionias.in ©Vision IAS

Google it:- https://upscpdf.com


https://t.me/UPSC_PDF Download From > https://upscpdf.com https://t.me/UPSC_PDF

63. Consider the following: 66. 'Special 301 Report', often seen in news,
1. This period is referred to as the period of pertains to :
population explosion in India. (a) Global network of cities to fostering the
exchange of experience and mutual
2. It is marked by a rapid fall in mortality
learning towards sustainable living.
rate but high fertility in the population.
(b) Countries that are supporting terror
3. The average annual population growth financing and money laundering
rate observed in this period was high activities.
around 2%. (c) Trading partners of USA that do not
Which of the following time periods in adequately enforce Intellectual Property
Indian history is characterized by the above Rights or deny market access to USA
(d) WTO's prescribed norms on rules of
conditions?
origin and government procurement of
(a) 1901-1921
products.
(b) 1921-1951
(c) 1951-1981 67. In a given time period, which of the
(d) 1981-2020 following best describes the term 'Actual
Growth of Population' of a given region?
64. Consider the following statements: (a) It is the total increase in the number of
1. Primitive human society was highly new births.
(b) It is the net increase in the population of
influenced by the strong forces of nature.
a region after accounting only for new
2. Level of technological development was
births and deaths.
low and humans were afraid of fury of (c) It refers to the change in number of
nature. inhabitants of a region due to births,
3. Ellen Semple was a proponent of this deaths as well as migration.
school of Human Geography. (d) It refers to the change in the fertility
Which of the following schools of Human rates of a population.

Geography do the above statements refer to?


68. Consider the following statements in the
(a) Possibilism
context of 'Public Utility Services' (PUS):
(b) Environmental Determinism
1. These are the businesses engaged in
(c) Cultural Determinism supplying essential goods and/or
(d) Neo Determinism services of the daily necessity for the
general public.
65. What is common to the sites of Mayurbhanj, 2. Essential Commodities Act 1955
Kudremukh and Bailadila? empowers the Government to declare an
industry as a public utility service.
(a) They are important oil fields.
Which of the statements given above is/are
(b) They are iron ore belts in India.
correct?
(c) They are well known sites of nuclear (a) 1 only
power plants. (b) 2 only
(d) They are one of the largest sites of (c) Both 1 and 2
copper mines. (d) Neither 1 nor 2

12 www.visionias.in ©Vision IAS

Google it:- https://upscpdf.com


https://t.me/UPSC_PDF Download From > https://upscpdf.com https://t.me/UPSC_PDF

69. Consider the following pairs in the context 72. Arrange the following Executive Capital
of shifting cultivation : cities of the states of India from north to
Local name Region south:
1. Milpa : India 1. Bengaluru
2. Vishakhapatnam
2. Taungiya : Myanmar
3. Raipur
3. Ladang : Malaysia
4. Thiruvananthapuram
Which of the pairs given above are correctly
Select the correct answer using the code
matched?
given below.
(a) 1 and 2 only
(a) 3-2-4-1
(b) 2 and 3 only (b) 3-2-1-4
(c) 1 and 3 only (c) 2-3-1-4
(d) 1, 2 and 3 (d) 2-3-4-1

70. With reference to the Regional 73. Consider the following statements regarding
Comprehensive Economic Partnership the differences between growth and
(RCEP), consider the following statements: development:

1. It is a proposed agreement between 1. While growth can be positive or


negative development can never be
member states of ASEAN and its free
negative.
trade agreement (FTA) partners.
2. While growth is a quantitative change,
2. The pact aims to cover trade in goods as
development is a qualitative change.
well as services.
3. Positive growth always leads to
Which of the statements given above is/are development.
correct? Which of the statements given above is/are
(a) 1 only correct?
(b) 2 only (a) 1 and 2 only
(c) Both 1 and 2 (b) 2 and 3 only
(d) Neither 1 nor 2 (c) 1, 2 and 3
(d) 2 only

71. Which of the following factors influence the


74. Natural gas reserves fields can be found in
location of industries?
which of the following states of India?
1. Access to labor supply
1. Andhra Pradesh
2. Government policy
2. Tripura
3. Access to sources of energy
3. Rajasthan
Select the correct answer using the code
Select the correct answer using the code
given below. given below.
(a) 1 and 3 only (a) 2 and 3 only
(b) 1 and 2 only (b) 1 and 3 only
(c) 2 and 3 only (c) 1 and 2 only
(d) 1, 2 and 3 (d) 1, 2 and 3
13 www.visionias.in ©Vision IAS

Google it:- https://upscpdf.com


https://t.me/UPSC_PDF Download From > https://upscpdf.com https://t.me/UPSC_PDF

75. Which of the following is/are considered 78. Consider the following statements:
'Urban Agglomerations'? 1. In India, sex ratio is calculated as the
1. A town and its adjoining urban
number of females per thousand males
outgrowths.
2. Two or more contiguous towns with or in the population.
without their outgrowths. 2. The current average global sex ratio is in
3. A city and one or more adjoining towns favour of females.
with their outgrowths together forming a
Which of the statements given above is/are
contiguous spread.
Select the correct answer using the code correct?
given below. (a) 1 only
(a) 3 only (b) 2 only
(b) 1 and 2 only
(c) Both 1 and 2
(c) 2 and 3 only
(d) 1, 2 and 3 (d) Neither 1 nor 2

76. With reference to population statistics, 79. Global Forest Resource Assessment 2020,
which of the following statements is not recently seen in news, has been published by
correct?
:
(a) The population of India is larger than the
total population of North America. (a) Global Green Growth Institute
(b) The population density of India has (b) International Panel on Climate Change
more than doubled since Independence. (c) United Nations Food and Agriculture
(c) West Bengal is the state with highest
Organisation
population density in India.
(d) Arunachal Pradesh has the lowest (d) United Nations Environment Program
density of population among the states in
India. 80. Consider the following statements about
nuclear mineral reserves:
77. Which of the following geographical factors
1. Thorium is more abundant than Uranium
have made the Steppe regions, ‘the granaries
of the world’? in Earth's Crust.
1. Cool and moist climate during the 2. India does not have any known reserves
spring season helps in the growth of of Uranium.
wheat.
3. Monazite and Ilmenite are important
2. Warm and sunny summer is
advantageous during the harvesting sources of thorium.
period. Which of the statements given above are
Select the correct answer using the code correct?
given below.
(a) 1 and 2 only
(a) 1 only
(b) 2 only (b) 2 and 3 only
(c) Both 1 and 2 (c) 1 and 3 only
(d) Neither 1 nor 2 (d) 1, 2 and 3
14 www.visionias.in ©Vision IAS

Google it:- https://upscpdf.com


https://t.me/UPSC_PDF Download From > https://upscpdf.com https://t.me/UPSC_PDF

81. In the context of economic geography, what 84. With reference to the recently launched 'One
are the Ports of Call? Nation, One Ration Card' system, consider
(a) Ports that act as collection centers where the following statements:
1. It aims to provide new micro-chip
the goods are brought from different
enabled ration cards to beneficiaries
countries for export. for purchasing food grains.
(b) Ports that are concerned with the 2. It enables existing beneficiaries under
transportation of passengers and mail the National Food Security Mission to
across water bodies covering short receive food grain from any Fair Price
distances. Shop in the country.
3. The Annavitran portal enables migrant
(c) Ports which serve warships and have
workers and their family members to
repair workshops for them. avail PDS benefits outside their home
(d) Ports developed on main sea routes state.
where ships anchor for refueling, Which of the statements given above is/are
watering and taking food items. correct?
(a) 2 only
(b) 1 and 3 only
82. Consider the following statements:
(c) 2 and 3 only
1. Most of the petroleum occurrences in
(d) 1, 2 and 3
India are associated with anticlines and
fault traps in the rock formations of the 85. Which of the following industries are most
tertiary age. likely to be located near their raw material
2. India has only field-based refineries and producing areas?
not marked based ones. 1. Pulp Industries
2. Copper Smelting
Which of the statements given above is/are
3. Pig Iron Industries
correct?
4. Synthetic Nitrogen Manufacturing
(a) 1 only Industries
(b) 2 only Which of the statements given above is/are
(c) Both 1 and 2 correct?
(d) Neither 1 nor 2 (a) 1 and 2 only
(b) 1, 3 and 4 only
(c) 1, 2 and 3 only
83. Consider the following statements:
(d) 2 and 4 only
1. Cropping Intensity in India has remained
below 1 for the last seven decades. 86. Consider the following statements about
2. The net sown area as a percentage of the mixed farming:
total cultivable area has decreased over 1. Animal husbandry is an important part
the last seven decades. of mixed farming.
2. It involves high capital expenditure and
Which of the statements given above
extensive use of chemical fertilizers.
is/are correct?
Which of the statements given above is/are
(a) 1 only correct?
(b) 2 only (a) 1 only
(c) Both 1 and 2 (b) 2 only
(d) Neither 1 nor 2 (c) Both 1 and 2
(d) Neither 1 nor 2
15 www.visionias.in ©Vision IAS

Google it:- https://upscpdf.com


https://t.me/UPSC_PDF Download From > https://upscpdf.com https://t.me/UPSC_PDF

87. Consider the following statements: 90. Consider the following statements in the
1. Oases are depressions of varying sizes in context of Energy Transition Index:
desert regions. 1. The index is published by the
2. Groundwater is the main source of water
International Energy Agency to identify
in oases.
the speed and direction of energy
3. Oases help in supporting a settled life in
desert regions. transition globally.
Which of the statements given above is/are 2. In the index, India has found a place
correct? among the top hundred countries.
(a) 1 only Which of the statements given above is /are
(b) 1, 2 and 3 correct?
(c) 3 only
(a) 1 only
(d) 2 and 3 only
(b) 2 only
(c) Both 1 and 2
88. Consider the following pairs:
Tribes Dwellings (d) Neither 1 nor 2

1. Bindibu : Zeriba
2. Tuaregs : Wurlies 91. Which of the following conditions are
3. Gobi Mongols : Yurts favorable for the cultivation of Tea?
Which of the pairs given above is/are 1. Moderate Temperatures.
correctly matched? 2. Medium Rainfall
(a) 1 and 2 only
3. Well-drained highland slopes.
(b) 2 and 3 only
Select the correct answer using the code
(c) 3 only
(d) 1, 2 and 3 given below.
(a) 1 and 2 only
89. According to the revised definitions of (b) 2 and 3 only
MSMEs, which of the following statements (c) 1 and 3 only
(criteria) given below are correct? (d) 1, 2 and 3
1. The distinction between the
manufacturing and service sector to be
92. Which of the following are pass that
eliminated.
connects India and Nepal?
2. The maximum annual turnover limit for
1. Lipulekh
medium-scale industries is capped at 250
crores. 2. Nathu La
3. The maximum investment limit for 3. Jelep La
medium-scale industries is capped at 50 4. Zoji La
crores. Select the correct answer using the codes
Select the correct answer using the code given below.
given below.
(a) 1 and 2 only
(a) 2 and 3 only
(b) 1, 3 and 4 only
(b) 1 and 3 only
(c) 1 and 2 only (c) 1 only
(d) 1, 2 and 3 (d) 2, 3 and 4 only
16 www.visionias.in ©Vision IAS

Google it:- https://upscpdf.com


https://t.me/UPSC_PDF Download From > https://upscpdf.com https://t.me/UPSC_PDF

93. Arrange the following continents in 96. Daimer Basha Dam, recently seen in the
increasing order of their population density. news, is a

1. Asia (a) Joint venture between China and


Pakistan in the Gilgit-Baltistan region.
2. Africa
(b) Strategic dam built under China-
3. Europe
Bangladesh joint agreement in
Select the correct answer using the code
Chittagong province.
given below. (c) Dam built in East Africa under the India-
(a) 2-1-3 Japan Asia- Africa economic corridor
(b) 2-3-1 project.
(c) 1-2-3 (d) Dam built by China on upper stretches

(d) 3-2-1 of river Tsangpo.

97. Consider the following statements with


94. Consider the following statements about
regard to the Samadhan challenge:
methane hydrates:
1. It is launched to test the ability of govt
1. They are trapped ice-like structures
officials to tackle the COVID-19
containing 90-95% of methane in it. pandemic.
2. They are found in permafrost tundra 2. Government Departments and Private
basins in Canada and Russia. organizations will compete to provide
3. Seabed volcanic ash sediments in innovative ideas on Public service

Andaman are an important source of delivery.


3. It is launched by the Ministry of Home
Methane Hydrates.
Affairs.
Which of the statements given above is/are
Which of the statements given above
correct?
is/are not correct?
(a) 1 and 2 only (a) 1 and 3 only
(b) 1 and 3 only (b) 3 only
(c) 2 and 3 only (c) 2 only
(d) 1, 2 and 3 (d) 1, 2 and 3

95. Which of the following states in India is 98. Y is a housekeep. She does cooking and
cleaning the house on a daily basis. Y also
setting up the first observatory for
serves the elderly and look after them. Y is
mammals?
engaged in which sectors of the economy?
(a) Odisha
(a) Primary
(b) Uttar Pradesh
(b) Tertiary
(c) Bihar (c) Quaternary
(d) West Begal (d) Quinary
17 www.visionias.in ©Vision IAS

Google it:- https://upscpdf.com


https://t.me/UPSC_PDF Download From > https://upscpdf.com https://t.me/UPSC_PDF

99. Consider the following statements with


respect to the National Waterway 1:
1. It runs from Haldia to Prayagraj across
the Ganges, Bhagirathi, and Hooghly
river system.
2. It passes through six states.
3. It is the longest waterway in India.
Which of the statements given above is/are
correct?
(a) 1 only
(b) 1 and 2 only
(c) 1 and 3 only
(d) 3 only

100. The Indian National Super Model, to


monitor the transmission of COVID-19
infection is recently launched by which of
the following ministries?
(a) Ministry of Home Affairs
(b) Ministry of Health and Family Welfare
(c) Ministry of Science and Technology
(d) Ministry of Statistics and Program
Implementation

18 www.visionias.in ©Vision IAS

Google it:- https://upscpdf.com


https://t.me/UPSC_PDF Download From > https://upscpdf.com https://t.me/UPSC_PDF

VISION IAS
www.visionias.in
ANSWERS & EXPLANATIONS
GENERAL STUDIES (P) TEST – 3181 (2021)

Q 1.C
• A market garden is the relatively small-scale production of fruits, vegetables and flowers as cash crops,
frequently sold directly to consumers and restaurants.
• In north-western Europe intensive market gardening is carried out in many specialized areas where
climatic, soil, small size of the farms and other factors best suit this form of agriculture.
• This region includes some of the most crowded parts of the globe and industrialized nations. Infact
there are more towns and cities than in other continents despite its small size. Most of its produce is
consumed in the domestic market. So, it has little surplus to export. It is, in fact, a net importer of food
crops and other horticultural products. Hence, statement 1 is correct.
• As most of the horticulture crops are perishable, a good network of transport is indispensable. The
produce is conveyed by high speed conveyances such as trucks or vans and is so called as truck farming in
the USA. In Australia, high speed boats ply across the Bass Strait daily from Tasmania to most of the
large cities in mainland Australia. Hence, statement 2 is correct.
Q 2.D
• The Austric languages of India belong to the Austro-Asiatic sub-family, which are represented by
languages of the Munda or Kol Group, spoken in the central, eastern and north-eastern India and
languages of the Mon-Khmer group like Khasi and Nicobarese. These are very ancient languages which
have been in existence much before the advent of Aryans.
• Dravidian is the second most important group and comprises mainly of languages spoken in Southern
India. It covers about 25% of the Indian population.
• The Sino-Tibetan or Mongoloid family stretches all over the sub-Himalayan tracts, covering North Bihar,
North Bengal, Assam up to the north-eastern frontiers of the country.
• Indo-European language has the highest number of speakers in India. It is the biggest of the language
groups in India and accounts for about 73% of the total Indian population. It comprises of all the principal
languages of northern and western India such as Hindi, Bengali, Marathi, Gujarati, Punjabi, Sindhi,
Rajasthani, Assamese, Oriya, Pahari, Bihari, Kashmiri, Urdu and Sanskrit. Hence option (d) is the
correct answer.

1 www.visionias.in ©Vision IAS

Google it:- https://upscpdf.com


https://t.me/UPSC_PDF Download From > https://upscpdf.com https://t.me/UPSC_PDF

Q 3.A
• The sugar industry is the second most important agro-based industry in the country. Brazil is the top
country with sugar cane production in the world. As of 2018, sugar cane production in Brazil was
746 million tonnes that account for 39.20% of the world's sugar cane production. The top 5
countries (others are India, China, Thailand, and Pakistan) account for 73.70% of it. Hence
statement 2 is not correct.
• India accounts for about 19 percent of the world's production of sugarcane. But it occupies only 2.4
percent of total cropped area in the country.
• This industry provides employment for more than 4 lakh persons directly and a large number of farmers
indirectly. The sugar industry is a seasonal industry because of the seasonality of raw materials.
Hence statement 1 is correct.
• The development of the industry on modern lines dates back to 1903 when a sugar mill was started in
Bihar. Subsequently, sugar mills were started in other parts of Bihar and Uttar Pradesh. In 1950-51, 139
factories were in operation. The number of sugar factories rose to 662 in 2010-11.
• Sugarcane is a weight-losing crop. The ratio of sugar to sugarcane varies between 9 to 12 percent
depending on its variety. Its sucrose content begins to dry during haulage after it has been harvested from
the field. Better recovery of sugar is dependent upon its being crushed within 24 hours of its harvesting.
Sugar factories hence are located within the cane producing regions. Maharashtra has emerged as a
leading sugar producer in the country and produces more than one-third of the total production of
the sugar in the country
• Uttar Pradesh is the second-largest producer of sugar. The sugar factories are concentrated in two belts –
the Ganga-Yamuna doab and the Tarai region. The major sugar-producing centers in the Ganga -Yamuna
doab are Saharanpur, Muzaffarnagar, Meerut, Ghaziabad, Baghpat, and Bulandshahr districts; while Kheri
Lakhimpur, Basti, Gonda, Gorakhpur, Bahraich are important sugar-producing districts in the Tarai
region.
• The crop yield is low in Northern India than in Southern India. Uttar Pradesh's yield is low, but in
Maharashtra, Karnataka, and Tamil Nadu the yield is high. Hence statement 3 is not correct.

Q 4.D
• Cotton is a “pure” raw material that does not lose weight in the manufacturing process. So other
factors, like, the power to drive the looms, labor, capital, or market may determine the location of the
industry. At present, the trend is to locate the industry at or close to markets, as it is the market that
decides what kind of cloth is to be produced. Also, the market for the finished products is extremely
variable, therefore, it becomes important to locate the mills close to the market.
• The cotton textile industry is one of the traditional industries of India. In the ancient and medieval times, it
used to be only a cottage industry. India was famous worldwide for the production of muslin, a very fine
variety of cotton cloth, calicos, chintz, and other different varieties of fine cotton cloth. The development
of this industry in India was due to several factors. One, it is a tropical country and cotton is the most
comfortable fabric for a hot and humid climate. Second, a large quantity of cotton was grown in India.
Abundant skilled labor required for this industry was available in this country. In fact, in some areas, the
people were producing cotton textiles for generations and transferred the skill from one generation to the
other and in the process perfected their skills.
• There are three cotton-growing areas, i.e. parts of Punjab, Haryana and northern Rajasthan in the north-
west, Gujarat and Maharashtra in the west and plateaus of Andhra Pradesh, Karnataka and Tamil Nadu in
the south. Leading producers of this crop are Gujarat, Maharashtra and Telangana per hectare
output of cotton is high under irrigated conditions in the north-western region of the country. Its
yield is very low in Maharashtra where it is grown under rainfed conditions.
• Initially, the British did not encourage the development of the indigenous cotton textile industry. They
exported raw cotton to their mills in Manchester and Liverpool and brought back the finished products to
be sold in India. This cloth was cheaper because it was produced at mass scale in factories in the U.K. as
compared to the cottage based industries of India. In 1854, the first modern cotton mill was established
in Mumbai.
• All the statements are correct. Hence the correct answer is option (d).

2 www.visionias.in ©Vision IAS

Google it:- https://upscpdf.com


https://t.me/UPSC_PDF Download From > https://upscpdf.com https://t.me/UPSC_PDF

Q 5.C
• About Prana-Vayu: It is a low-cost portable ventilator. It is a closed-loop ventilator which can deliver
the required amount of air to the patient, with an automated process controlling the pressure and
flow rates. Hence option (c) is the correct answer.
o It is also featured with feedback that can control tidal volume and the number of breaths per minute.
o It will be useful for a wide degree of congestions in the respiratory tract and is applicable for all age
group patients, especially the elderly.
o It has a specific characteristic that it does not require compressed air for functioning and can be
especially useful in cases when hospital wards or open areas are converted into ICUs.

Q 6.A
• Quinary activities are services that focus on the creation, re-arrangement and interpretation of new and
existing ideas; data interpretation and the use and evaluation of new technologies.
• Often referred to as ‘gold collar’ professions, they represent another subdivision of the tertiary sector
representing special and highly paid skills of senior business executives, government officials, research
scientists, financial and legal consultants, etc. Hence option (a) is the correct answer.
• The economic activities of the people of the world related to the production, distribution, exchange, use,
and service related to the employment of various livelihoods for the purpose of livelihood are called
economic activities. Through different economic activities, people are separated from their clothes and
colors. They are:
o Red-Collar Worker: The employees at the primary level are called Red-Collar Worker. Because
they have to work in the sun and with water all the time outside the house. For example, farmer or
farming staff.
o Blue-Collar Worker: The workers employed in the secondary level are called blue-collar workers.
For example, workers engaged in factories or industries.
o Black-Collar Worker: The workers of coal and mining oil extraction are called Black-Collar
Worker. In some cases, it is used to associate black marketing activities.
o Pink-Collar Worker: Employees employed in tertiary activities are called Pink-Collar Worker.
Example: Nurses, Secretaries, Elementary School Teacher etc.
o White-Collar Worker: A white-collar worker is a person who performs professional, desk,
managerial, or administrative work. White-collar work may be performed in an office or other
administrative setting. Some examples include: corporate executives, advertising and public relation
professionals, architects, stockbrokers, doctors, dentists and dietitians.
o Yellow-Collar Worker: People who are involved in creative work, who also do works of creating
blue-collar and white-collar work outside the chardeal, they are called 'Yellow-Collar Worker'.Eg-
photographers, filmmakers, directors, editors etc.
o Green-Collar Worker: Natural elements (solar, water, air, etc.) are the 'Green-Collar Worker'
workers employed in the economic field. Eg-Solar panels, workers working in Greenpeace, World
wide fund for nature
o Orange-Collar Worker: They have been named after working for orange jump suits after the orange
clothes dressed by the residents of the Prison Labors.
o Gray-Collar Worker: Retired workers are known as 'Gray-Collar Worker'. That is, those who are
re-employed after any work from their work are called 'Gray-Collar Worker' and that work is called
Gray-Collar Work. E.g.- fire fighters, police officers, health-care professional, security guards etc.

Q 7.B
• Statement 1 is not correct: The doubling time is time it takes for a population to double in size. As can
be seen from the graph below, the population of the world has been doubling at a faster pace, reducing the
doubling in the last one century.
• Statement 2 is correct: As seen in the image given below, at the beginning of the twentieth century
(1900s), the population of the world was less than 2 billion whereas the current world population stands at
7 billion. Hence, at the beginning of the twentieth century, the world population was less than half of its
current levels.

3 www.visionias.in ©Vision IAS

Google it:- https://upscpdf.com


https://t.me/UPSC_PDF Download From > https://upscpdf.com https://t.me/UPSC_PDF

Q 8.A
• Recent Context: The Computer Emergency Response of Team (CERT) of India has issued warning
against a new malware called EventBot. The malware according to CERT steals personal financial
information from Android phone users. Hence option (a) is correct.
• The Eventbol malware that is spreading is Trojan. It cheats victims secretly attacking a computer or phone
operating system.
• Currently, the malware is capable of targeting more than 200 different financial applications.
• The malware has already looted cryptocurrency wallets, bank accounts in Europe and the US. It targets
money-transfer services, financial applications.

Q 9.A
• Statement 1 is not correct: India has 26 sedimentary basins covering an area of 3.14 million square
kilometers. The sedimentary basins of India, on land and offshore up to the 200m isobath, have an
areal extent of about 1.79 million sq. km. In the deepwater beyond the 200m isobath, the sedimentary area
has been estimated to be about 1.35 million sq. km. Thus, the total works out to 3.14 million sq. km. They
are spread across onland, offshore and deepwater areas.
• Statement 2 is correct: Over the last twelve years, there have been significant forward steps in exploring
the hydrocarbon potential of the sedimentary basins of India. The unexplored area has come down to
15% which was 50% in 1995-96. They are an important source of Natural Gas, CBM and Shale Gas
along with Oil, Petroleum.
• The sedimentary basins of the country have been classified into four categories as below:
o Category-I: Basins with established commercial production. Cambay, Mumbai Offshore, Rajasthan,
Krishna Godavari, Cauvery, Assam Shelf and Assam-Arakan Fold Belt
o Category-II: Basins with known accumulation of hydrocarbons but no. commercial production
achieved so far. Kutch, Mahanadi-NEC (North East Coast) Basin,Andaman-Nicobar, Kerala-Konkan-
Lakshadweep Basin.
o Category-III: Basins having hydrocarbon shows that are considered geologically prospective.
Himalayan Foreland Basin, Ganga Basin, Vindhyan basin, Saurashtra Basin,Kerela Konkan Basin,
Bengal Basin.
o Category-IV: Basins having uncertain potential which may be prospective by analogy with similar
basins in the world. Karewa basin, Spiti-Zanskar basin, Satpura–South Rewa–Damodar basin,
Chhattisgarh Basin, Narmada basin, Deccan Syneclise, Bhima-Kaladgi, Bastar Basin,
PranhitaGodavari basin, Cuddapah basin.

4 www.visionias.in ©Vision IAS

Google it:- https://upscpdf.com


https://t.me/UPSC_PDF Download From > https://upscpdf.com https://t.me/UPSC_PDF

Q 10.B
• Statement 1 is not correct and statement 2 is correct: Agriculture accounts for most of the surface and
groundwater utilization, it accounts for 89 percent of the surface water and 92 percent of the groundwater
utilization. While the share of the industrial sector is limited to 2 percent of the surface water utilization
and 5 percent of the ground-water, the share of the domestic sector is higher (9 percent) in surface water
utilization as compared to groundwater. The share of the agricultural sector in total water utilization is
much higher than in other sectors.
• Rain-fed areas contributed significantly to the country’s food production. They account for 89 percent of
millet production, 88 percent of pulses, 73 percent of cotton, 69 percent of oilseeds, and 40 percent rice
production in the country. Besides, rain-fed areas support 64 percent of cattle, 74 percent of sheep, and 78
5 www.visionias.in ©Vision IAS

Google it:- https://upscpdf.com


https://t.me/UPSC_PDF Download From > https://upscpdf.com https://t.me/UPSC_PDF

percent of the goat population in the country. About 61 percent of India’s farmers rely on rain-fed
agriculture and 55 percent of the gross cropped area is under rain-fed farming. India ranks first in
rain-fed agriculture, both in the area and the value of produce.

Q 11.D
• In India compact or clustered village of a few hundred houses is a rather universal feature, particularly in
the northern plains. However, there are several areas, which have other forms of rural settlements. There
are various factors and conditions responsible for having different types of rural settlements inIndia.
These include:
o physical features – nature of terrain, altitude, climate and availability of water
o cultural and ethnic factors – social structure, caste and religion
o security factors – defence against thefts and robberies
• Hence all the options are correct.

Q 12.B
• On the basis of chemical and physical properties, minerals may be grouped under two main categories of
metallics and non-metallics. Metallic minerals are the sources of metals. Iron ore, copper, gold produce
metal and are included in this category. Metallic minerals are further divided into ferrous and non-ferrous
metallic minerals. All those minerals which have iron content are ferrous such as iron ore itself and those
which do not have iron content are non-ferrous such as copper, bauxite, etc. Hence options 1 and 3 are
correct.
• Non-metallic minerals are either organic in origins such as fossil fuels also known as mineral fuels which
are derived from the buried animal and plant life such as coal and petroleum. Other types of non-
metallic minerals are inorganic in origin such as mica, limestone, and graphite, etc. Hence options 2
and 4 are not correct.
• Metallic minerals are found in igneous and metamorphic rocks whereas non-metallic minerals have a
sedimentary origin.

Q 13.C
• The new Industrial Policy was announced in 1991. The major objectives of this policy were to build on
the gains already made, correct the distortions or weaknesses that have crept in, maintain a sustained
growth in productivity and gainful employment, and attain international competitiveness.
• Within this policy, measures initiated were: abolition of industrial licensing, free entry to foreign
technology, foreign investment policy, access to the capital market, open trade, abolition of phased
manufacturing program, and liberalized industrial location program.
• The policy has three main dimensions: liberalization, privatization, and globalization.
• The industrial licensing system has been abolished for all except six industries related to security,
strategic or environmental concerns.
• At the same time, the number of industries reserved for the public sector since 1956 has been reduced
from 17 to 4.
• Industries related to atomic energy, substances specified in the Schedule of the Department of
Atomic Energy as well as Railways have remained under the public sector. Hence, statement 1 is
correct.
6 www.visionias.in ©Vision IAS

Google it:- https://upscpdf.com


https://t.me/UPSC_PDF Download From > https://upscpdf.com https://t.me/UPSC_PDF

• The government also has decided to offer a part of the shareholdings in the public enterprises to financial
institutions, the general public, and workers.
• The threshold limits of assets have been scrapped and no industry requires prior approval for
investing in the delicensed sector. They only need to submit a memorandum in the prescribed
format. Hence, statement 2 is correct.
• The industrial policy has been liberalized to attract private investors both domestic and multi-nationals.
New sectors like mining, telecommunications, highway construction, and management have been thrown
open to private companies.

Q 14.C
• Direct Seeding of rice (DSR) : Punjab and Haryana could face a shortage of labourers to undertake
transplantation of paddy in upcoming kharif season.
• This now has encouraged them to adopt DSR in place of conventional transplanting.
• DSR refers to process of establishing a rice crop from seeds sown in field rather than by transplanting
seedlings from nursery. Hence statement 1 is correct.
• In transplanting, farmers prepare nurseries where paddy seeds are first sown and raised into young
plants. These seedlings are then uprooted and replanted 25-35 days later in main field with standing
water.
o It ensures a uniform plant stand and gives rice crop a head start over emerging weeds.
o In this, standing water acts as herbicide and prevents growth of weeds by denying them oxygen in
submerged stage.
• In DSR, water is replaced by real chemical herbicides.
• Advantages of DSR:
o saving irrigation water
o Saves labour, energy, time,
o Reduces emission of greenhouse-gases, etc. Hence statement 2 is correct.
• Disadvantage of DSR:
o high weed infestation,
o evolution of weedy rice,
o increase in soil borne pathogens (nematodes),
o nutrient disorders,
o exposure to birds and rats incidence,
o rice blast, brown leaf spot etc.

Q 15.B
• Shale gas & oil is defined as natural gas & oil from shale formations. Shale gas is an unconventional
source of energy found in non-porous rocks. The shale acts as both the source and the reservoir for these
unconventional hydrocarbons. Older shale wells were vertical while more recent wells are primarily
horizontal and need artificial stimulation, like hydraulic fracturing, to produce. Only shale formations with
certain characteristics will produce gas and oil.
• The Government of India has carried out studies through various national and international agencies for
the identification of shale oil and gas resources in the country. Based on the data available from
conventional oil/gas exploration in the country for the last so many years, the country holds promising
reserves of Shale Gas & Oil resources and the following sedimentary basins are considered prospective
from Shale oil and gas point of view:
o Cambay Basin
o Gondwana Basin
o KG Basin
o Cauvery Basin
o Indo-Gangetic Basin
o Assam-Arakan Basin
• Hence only options 1, 2 and 3 are correct.

Q 16.C
• Recent context: In the wake of several complaints regarding home quarantined persons moving around
without following restrictions, thereby increasing the risk of spreading COVID-19 infection, the
Karnataka government is mulling over the idea of “geofencing” such patients.
• Geofencing: It is a location-based service in which an app or other software uses GPS, RFID, Wi-Fi
or cellular data to trigger a pre-programmed action. Hence option (c) is the correct. answer.
7 www.visionias.in ©Vision IAS

Google it:- https://upscpdf.com


https://t.me/UPSC_PDF Download From > https://upscpdf.com https://t.me/UPSC_PDF

• When a mobile device or RFID tag enters or exits a virtual boundary set up around a geographical
location, known as a geo-fence.
• The geo-fencing is accurate by up to 300 m. To make use of geofencing, an administrator or developer
must first establish a virtual boundary around a specified location in GPS- or RFID-enabled software. This
virtual defense will then trigger a response. When an authorized device enters or exits that area, as
specified by the administrator or developer.

Q 17.C
• Nuclear energy has emerged as a viable source in recent times. Important minerals used for the generation
of nuclear energy are uranium and thorium.
• Uranium deposits occur in the Dharwar rocks.
• Geographically, uranium ores are known to occur in several locations along the Singbhum Copper belt.
• It is also found in Udaipur, Alwar, and Jhunjhunu districts of Rajasthan, Durg district of Chhattisgarh,
Bhandara district of Maharashtra, and Kullu district of Himachal Pradesh.
• Thorium is mainly obtained from monazite and ilmenite in the beach sands along the coast of Kerala and
Tamil Nadu.
• World’s richest monazite deposits occur in Palakkad and Kollam districts of Kerala, near Vishakhapatnam
in Andhra Pradesh and Mahanadi river delta in Odisha.
• Atomic Energy Commission was established in 1948, progress could be made only after the establishment
of the Atomic Energy Institute at Trombay in 1954 which was renamed as the Bhabha Atomic Research
Centre in 1967.
• The important nuclear power projects are Tarapur (Maharashtra), Rawatbhata near Kota
(Rajasthan), Kalpakkam (Tamil Nadu), Narora (Uttar Pradesh), Kaiga (Karnataka) and
Kakarapara (Gujarat). Hence only pairs 1 and 2 are correctly matched.

Q 18.B
• The first stage has high fertility and high mortality because people reproduce more to compensate for the
deaths due to epidemics and variable food supply. The population growth is slow and most of the people
are engaged in agriculture where large families are an asset. Life expectancy is low, people are mostly
illiterate and have low levels of technology. Two hundred years ago all the countries of the world were in
this stage.
8 www.visionias.in ©Vision IAS

Google it:- https://upscpdf.com


https://t.me/UPSC_PDF Download From > https://upscpdf.com https://t.me/UPSC_PDF

• Fertility remains high in the beginning of second stage but it declines with time. This
is accompanied by a reduced mortality rate. Improvements in sanitation and health conditions lead
to decline in mortality. Because of this gap, the net addition to the population is high. In the last
stage, both fertility and mortality decline considerably. The population is either stable or grows slowly.
The population becomes urbanised, literate and has high technical knowhow and deliberately controls the
family size.

Q 19.C
• The dispersed or isolated settlement pattern in India appears in the form of isolated huts or hamlets of
few huts in remote jungles, or on small hills with farms or pasture on the slopes. Extreme dispersion of
settlement is often caused by the extremely fragmented nature of the terrain and land resource base of
habitable areas. Many areas of Meghalaya, Uttaranchal, Himachal Pradesh and Kerala have this type of
settlement. Hence pair 1 is correctly matched.
• The clustered rural settlement is a compact or closely built-up area of houses. In this type of village, the
general living area is distinct and separated from the surrounding farms, barns and pastures. The closely
built-up area and its intervening streets present some recognisable pattern or geometric shape, such
as rectangular, radial, linear, etc. Such settlements are generally found in fertile alluvial plains and in the
northeastern states. Sometimes, people live in the compact village for security or defence reasons, such as
in the Bundelkhand region of central India and in Nagaland. In Rajasthan, scarcity of water has
necessitated compact settlement for maximum utilisation of available water resources. Hence pair 2 is
correctly matched.
• Hamleted Settlement: Sometimes a settlement is fragmented into several units physically separated from
each other bearing a common name. These units are locally called panna, para, palli, nagla, dhani, etc.
in various parts of the country. This segmentation of a large village is often motivated by social and ethnic
factors. Such villages are more frequently found in the middle and lower Ganga plain, Chhattisgarh
and lower valleys of the Himalayas. Hence pair 3 is correctly matched.

Q 20.B
• The World Health Assembly is the decision-making body of WHO.
• It is attended by delegations from all WHO Member States and focuses on a specific health agenda
prepared by the Executive Board. Hence statement 1 is correct.
• The main functions of the World Health Assembly are to
o determine the policies of the Organization
o appoint the Director-General, supervise financial policies
o review and approve the proposed program budget.
• The Health Assembly is held annually in Geneva, Switzerland. Hence statement 2 is correct.
• The 73rd session of the World Health Assembly (WHA) took place virtually.
• A resolution was brought forward by the European Union (EU) and was endorsed at the Assembly. It
asked for an “impartial, independent, and comprehensive evaluation” of the WHO's response to the
COVID-19 pandemic.
• The resolution also called for the identification of the “zoonotic” source of the coronavirus. The
origin of the virus is currently believed to be a wet market in Wuhan, China
9 www.visionias.in ©Vision IAS

Google it:- https://upscpdf.com


https://t.me/UPSC_PDF Download From > https://upscpdf.com https://t.me/UPSC_PDF

• Apart from this, the Assembly also addressed a global vaccine action plan with the “Immunisation
Agenda 2030”. This aims to ensure immunization for all age groups to prevent the spread of preventable
diseases and sustaining vaccine supplies.
• WHO Executive Board Executive gives effect to the decisions and policies of the World Health
Assembly, to advise it and generally to facilitate its work (not World Health Assembly). Hence,
statement 3 is not correct.

Q 21.A
• The regions where farmers specialise in vegetables, the farming is known as truck farming. Hence
option (a) is the correct answer.
• The distance of truck farms from the market is governed by the distance that a truck can cover overnight,
hence the name truck farming.

Q 22.D
• The development of the iron and steel industry opened the doors to rapid industrial development in India.
Almost all sectors of the Indian industry depends heavily on the iron and steel industry for its basic
infrastructure.
• The raw materials besides iron ore and coking coal, essential for iron and steel industry are limestone,
dolomite, manganese, and fire clay. All these raw materials are gross (weight losing), therefore, the
best location for the iron and steel plants is near the source of raw materials. Hence statement 1 is
not correct.
• In India, there is a crescent of the shaped region comprising parts of Chhattisgarh, Northern Odisha,
Jharkhand, and western West Bengal, which is extremely rich in high-grade iron ore, good quality coking
coal and other supplementing raw materials. This region supports the major steel plants.
• The Indian iron and steel industry consists of large integrated steel plants as well as mini steel mills. It
also includes secondary producers, rolling mills, and ancillary industries.
• New steel plants that were set up in the Fourth Plan period are away from the main raw material
sources. All three plants are located in South India. The Vizag Steel Plant, in Vishakhapatnam in
Andhra Pradesh, is the first port-based plant that started operating in 1992. Its port location is of
advantage. Apart from major steel plants, there are more than 206 units located in different parts of
the country. Most of these use scrap iron as their main raw material and process it in electric
furnaces. Hence statement 2 is not correct.

Q 23.A
• Statement 1 is correct: In Western countries, males outnumber females in rural areas and females
outnumber males in urban areas. In countries like Nepal, Pakistan and India the case is reverse. The
excess of females in urban areas of U.S.A., Canada and Europe is the result of influx of females from
rural areas to avail of the vast job opportunities. Farming in these developed countries is also highly
mechanised and remains largely a male occupation. By contrast the sex ratio in Asian urban areas
remains male-dominated due to the predominance of male migration.
• Statement 2 is not correct: It is also worth noting that in countries like India, female participation in
farming activity in the rural area is fairly high. However, females do not outnumber the males engaged in
the primary sector ( as primary sector includes activities such as mining).

10 www.visionias.in ©Vision IAS

Google it:- https://upscpdf.com


https://t.me/UPSC_PDF Download From > https://upscpdf.com https://t.me/UPSC_PDF

Q 24.B
• Statement 1 is not correct: The Human Development Index (HDI) is a summary measure of average
achievement in key dimensions of human development: a long and healthy life, being knowledgeable and
have a decent standard of living. The HDI is the geometric mean of normalized indices for each of the
three dimensions. The origins of the HDI are found in the annual Human Development Reports
produced by the Human Development Report Office of the United Nations Development
Programme (UNDP). It was started in 1991 as Human Development Report by United Nations
Development Programme (UNDP).
• Statement 2 is correct: The Human Development Index (HDI) provides a single index measure to
capture three key dimensions of human development: a long and healthy life, access to knowledge and a
decent standard of living.
• India is placed low at 129 in Human Development Index in 2019.

Q 25.D
• The warm, bright summers and cool, moist winters enable the cultivation of a wide range of citrus fruits
such as oranges, lemons, limes, citrons and grapefruit in this region. The Mediterranean lands account
for 70 percent of the world's export of citrus fruits. Hence, this region is known as the world's orchard
lands. Hence, statement 1 is correct.
• In the Mediterranean region, 85 percent of the grapes cultivated are used for the production of
wine and it amounts to three-quarters of the world production. Here, traditionally commercial
viticulture is practiced. The inferior grapes are preserved as dried grapes and exported. Hence, statement
2 is correct.
• The summers being mostly warm and dry, some pastures can be found in the mountainous areas with a
comparatively cooler climate that supports a few sheep, goats and sometimes cattle and transhumance are
widely practiced. Hence, statement 3 is correct.

Q 26.B
• Watershed management basically refers to efficient management and conservation of surface and
groundwater resources. Hence statement 1 is not correct.
• It involves the prevention of runoff and storage and recharge of groundwater through various methods like
percolation tanks, recharge wells, etc. Hence statement 2 is correct.
• However, in broad sense watershed management includes conservation, regeneration, and judicious use of
all resources – natural (like land, water, plants, and animals) and the human within a watershed.
• Watershed management aims at bringing about a balance between natural resources on the one hand and
society on the other. The success of watershed development largely depends upon community
participation. Hence statement 3 is correct.
11 www.visionias.in ©Vision IAS

Google it:- https://upscpdf.com


https://t.me/UPSC_PDF Download From > https://upscpdf.com https://t.me/UPSC_PDF

• The Central and State Governments have initiated many watershed development and management
programs in the country. Some of these are being implemented by nongovernmental organizations also.
• Haryali is a watershed development project sponsored by the Central Government which aims at
enabling the rural population to conserve water for drinking, irrigation, fisheries, and afforestation.
The Project is being executed by Gram Panchayats with people’s participation.
• Neeru-Meeru (Water and You) program (in Andhra Pradesh) and Arvary Pani Sansad (in Alwar,
Rajasthan) have taken up constructions of various water-harvesting structures such as percolation tanks,
dug out ponds (Johad), check dams, etc., through people’s participation. Tamil Nadu has made water
harvesting structures in the houses compulsory. No building can be constructed without making structures
for water harvesting.
• Watershed development projects in some areas have been successful in rejuvenating the
environment and economy. However, there are only a few success stories. In the majority of cases, the
program is still in its nascent stage. There is a need to generate awareness regarding the benefits of
watershed development and management among people in the country, and through this integrated water
resource management approach, water availability can be ensured on a sustainable basis.

Q 27.C
• Demographic transition theory can be used to describe and predict the future population of any area. The
theory tells us that population of any region changes from high births and high deaths to low births and
low deaths as society progresses from rural agrarian and illiterate to urban industrial and literate
society. These changes occur in stages which are collectively known as the demographic cycle. Hence
both statements are correct.
• The first stage has high fertility and high mortality because people reproduce more to compensate for the
deaths due to epidemics and variable food supply. The population growth is slow and most of the people
are engaged in agriculture where large families are an asset. Life expectancy is low, people are mostly
illiterate and have low levels of technology. Two hundred years ago all the countries of the world were in
this stage. Fertility remains high in the beginning of the second stage but it declines with time. This
is accompanied by reduced mortality rate. Improvements in sanitation and health conditions lead to
decline in mortality. Because of this gap the net addition to population is high. In the last stage, both
fertility and mortality decline considerably. The population is either stable or grows slowly. The
population becomes urbanized, literate and has high technical knowhow and deliberately controls the
family size.

Q 28.C
• Population Growth rate of various religion has come down in the last decade (2001-2011). Hindu
Population Growth rate slowed down to 16.76 % from previous decade figure of 19.92% while Muslim
witness sharp fall in growth rate to 24.60% (2001-2011) from the previous figure of 29.52 % (1991-2001).
Such sharp fall in population growth rate for Muslims didn't happened in the last 6 decades. Christian
Population growth was at 15.5% while Sikh population growth rate stood at 8.4%. The most educated and
wealthy community of Jains registered least growth rate in 2001-2011 with figure of just 5.4%.
• As per the adjoining figure, the correct order of given religions in increasing order of their population
is Muslims-Christians-Sikhs-Buddhists. Hence option (c) is the correct answer.

12 www.visionias.in ©Vision IAS

Google it:- https://upscpdf.com


https://t.me/UPSC_PDF Download From > https://upscpdf.com https://t.me/UPSC_PDF

Q 29.C
• The highest level of decision-makers or policymakers performs quinary activities. These are subtly
different from the knowledge-based industries that the quinary sector in general deals with. Hence
statement 1 is correct.
• Quinary activities are services that focus on the creation, re-arrangement, and interpretation of new and
existing ideas; data interpretation, and the use and evaluation of new technologies. Often referred to as
‘gold collar’ professions, they represent another subdivision of the tertiary sector representing
special and highly paid skills of senior business executives, government officials, research scientists,
financial and legal consultants, etc. Their importance in the structure of advanced economies far
outweighs their numbers.
• New trends in quinary services include knowledge processing outsourcing (KPO) and ‘home shoring’, the
latter as an alternative to outsourcing. The KPO industry is distinct from Business Process Outsourcing
(BPO) as it involves highly skilled workers. It is information-driven knowledge outsourcing. KPO enables
companies to create additional business opportunities. Examples of KPOs include research and
development (R and D) activities, e-learning, business research, intellectual property (IP) research,
the legal profession, and the banking sector. Hence statement 2 is correct.

Q 30.D
• Mediterranean Climate (Western Margin Climate): Entirely confined to the western portion of
continental masses, between 30° and 45° north and south of the equator.
• The basic reason for this type of climate is the shifting of the westerly wind belts.
o In summer, the Westerlies belt is shifted a little polewards. Rain bearing winds are therefore not likely
to reach the Mediterranean land.
o The Mediterranean lands receive most of their precipitation in winter when the Westerlies shift
equator wards.
• Hence it is called ‘winter rain climate’. The rain comes in heavy showers and only on a few days with
bright sunny periods between them. This is another characteristic feature of the Mediterranean winter
rain.
• Spatial spread: Regions around the Mediterranean Sea, central Chile. California,the south-western tip
of Africa, southern Australia, and south-west Australia (Swanland).
• Orchard farming:
• The Mediterranean lands are also known as the world’s orchard lands.
• The fruit trees have long roots to draw water from considerable depths during the long summer
drought. Hence, statement 2 is correct.
• The thick, leathery skin of the citrus fruits prevents excessive transpiration. Hence, statement 3 is
correct.
• The long, sunny summer enables the fruits to be ripened and harvested. Hence, statement 1 is
correct.
• Olives, grapes, citrus and many nut trees like chestnuts, walnuts, hazelnuts and almonds are grown.

Q 31.A
• Most of the metallic minerals in India occur in the peninsular plateau region in the old crystalline rocks.
Over 97 percent of coal reserves occur in the valleys of Damodar, Sone, Mahanadi, and Godavari.
Petroleum reserves are located in the sedimentary basins of Assam, Gujarat and Mumbai High i.e. off-
shore region in the Arabian Sea. New reserves have been located in the Krishna-Godavari and Kaveri
basins. Most of the major mineral resources occur to the east of a line linking Mangaluru and
Kanpur. Minerals are generally concentrated in three broad belts in India. There may be some
sporadic occurrences here and there in isolated pockets. These belts are:
• The North-Eastern Plateau Region: This belt covers Chhotanagpur (Jharkhand), Odisha Plateau, West
Bengal, and parts of Chhattisgarh. It has a variety of minerals viz. iron ore coal, manganese, bauxite,
mica. Hence, pair 1 is correctly matched.
• The South-Western Plateau Region: This belt extends over Karnataka, Goa and contiguous Tamil
Nadu uplands and Kerala. This belt is rich in ferrous metals and bauxite. It also contains high-grade
iron ore, manganese, and limestone. This belt lacks in coal deposits except for Neyveli lignite. This
belt does not have as diversified mineral deposits as the north-eastern belt. Kerala has deposits of
monazite and thorium, bauxite clay. Goa has iron ore deposits. Hence, pair 3 is not correctly
matched. Anthracite Coal is found in parts of Jammu & Kashmir only.
• The North-Western Region: This belt extends along Aravali in Rajasthan and part of Gujarat and
minerals are associated with the Dharwar system of rocks. Copper, zinc has been major minerals.
13 www.visionias.in ©Vision IAS

Google it:- https://upscpdf.com


https://t.me/UPSC_PDF Download From > https://upscpdf.com https://t.me/UPSC_PDF

Hence pair 2 is correctly matched. Rajasthan is rich in building stones i.e. sandstone, granite,
marble. Gypsum and Fuller’s earth deposits are also extensive. Gujarat is known for its petroleum
deposits. The Himalayan belt is another mineral belt where copper, lead, zinc, cobalt, and tungsten are
known to occur.

Q 32.D
• Statement 1 is not correct: As shown in the table below, the decadal growth rate of the Indian population
has been declining since 1981.

• Statement 2 is not correct: As can be seen above, the population of India witnessed a decline in the year
1921 (1911) with -0.31% growth rate registered.
14 www.visionias.in ©Vision IAS

Google it:- https://upscpdf.com


https://t.me/UPSC_PDF Download From > https://upscpdf.com https://t.me/UPSC_PDF

Q 33.B
• The air services is better developed in the parts of Northern hemisphere mostly because of the high
demand for faster means of transportation due to the high density of population and economic
development.
• The Southern hemisphere lacks the demand aspect due to sparser population and low economic
development. Hence, statements 1 and 3 are correct.
• The Southern hemisphere is characterized by less continentality and the presence of large water bodies.
Hence, the construction of airstrips becomes difficult. Hence, statement 2 is not correct.
• Though the Southern hemisphere has less landmass characterized by forests, deserts and grasslands which
once cleared, construction of air service infrastructure is easily possible.

Q 34.A
• Industries are not evenly distributed in the country. They tend to concentrate on certain locations because
of the favorable locational factors.
• Major industrial regions of the country are given below:
o Mumbai-Pune Region
o Hugli Region
o Bengaluru-Tamil Nadu Region
o Gujarat Region
o Chotanagpur Region
o Vishakhapatnam-Guntur Region
o Gurugram-Delhi-Meerut Region, and
o Kollam-Thiruvananthapuram Region
• Minor Industrial Regions (13):
o Ambala-Amritsar
o Saharanpur-Muzzaffarnagar-Bijnor
o Indore-Dewas-Ujjain
o Jaipur-Ajmer
o Kolhapur-South Kannada
o Northern Malabar
o Middle Malabar
o Adilabad-Nizamabad
o Allahabad-Varanasi-Mirzapur
o Bhojpur-Munger
o Bilaspur-Korba
o Durg-Raipur
o Brahmaputra Valley

Q 35.B
• Transhumance: The process of migration from plain areas to pastures on mountains during
summers and again from mountain pastures to plain areas during winters is known as
transhumance.
• In mountain regions, such as Himalayas, Gujjars, Bakarwals, Gaddis and Bhotiyas migrate from plains to
the mountains in summers and to the plains from the high altitude pastures in winters. Hence option (b) is
the correct answer.
• Similarly, in the tundra regions, the nomadic herders move from south to north in summers and from
north to south in winters.
• The difference between transhumance and nomadic pastoralism is that true nomads follow an irregular
pattern of movement whereas transhumance has fixed seasonal patterns of movement. Both can herd
livestock in search of fresh pastures on which to graze.

Q 36.B
• Statement 1 is not correct: Pulses are a very important ingredient of vegetarian food as these are rich
sources of proteins. These are legume crops that increase the natural fertility of soils through nitrogen
fixation. India is a leading producer of pulses in the world. The cultivation of pulses in the country is
largely concentrated in the drylands of Deccan and central plateaus and northwestern parts of the
country. Pulses occupy about 11 percent of the total cropped area in the country. Being the rainfed
crops of drylands, the yields of pulses are low and fluctuate from year to year. Gram and tur are the main
pulses cultivated in India.
15 www.visionias.in ©Vision IAS

Google it:- https://upscpdf.com


https://t.me/UPSC_PDF Download From > https://upscpdf.com https://t.me/UPSC_PDF

• The oilseeds are produced for extracting edible oils. Drylands of Malwa plateau, Marathwada, Gujarat,
Rajasthan, Telangana, Rayalseema region of Andhra Pradesh, and Karnataka plateau are oilseeds growing
regions of India. These crops together occupy about 14 percent of the total cropped area in the
country. Groundnut, rapeseed and mustard, soybean, and sunflower are the main oilseed crops grown in
India.
• The coarse cereals together occupy about 16.50 percent of the total cropped area in the country.
• Statement 2 is not correct: The area under rice cultivation is the highest. Wheat is the second most
important cereal crop in India after rice. India produces about 12.3 percent of the total wheat production
of the world (2016).
• Statement 3 is correct: Cotton is a tropical crop grown in the Kharif season in semi-arid areas of the
country. India ranks second in the world in the production of cotton after China. Cotton occupies about
4.7 percent of the total cropped area in the country. There are three cotton-growing areas, i.e. parts of
Punjab, Haryana and northern Rajasthan in the north-west, Gujarat and Maharashtra in the west and
plateaus of Andhra Pradesh, Karnataka and Tamil Nadu in the south. Leading producers of this crop
are Gujarat, Maharashtra and Telangana per hectare output of cotton is high under irrigated conditions in
the north-western region of the country. Its yield is very low in Maharashtra where it is grown under
rainfed conditions.
• Jute is used for making coarse cloth, bags, sacks and decorative items. It is a cash crop in West Bengal
and adjoining eastern parts of the country. India lost large jute growing areas to East Pakistan
(Bangladesh) during partition. At present, India produces about three-fifth of jute production of the world.
West Bengal accounts for about three-fourths of the production in the country. Bihar and Assam are other
jute growing areas. Being concentrated only in a few states, this crop accounts for only about 0.5 per
cent of the total cropped area in the country.

Q 37.D
• The Global Snow Leopard and Ecosystem Protection Program (GSLEP) seeks to address high-
mountain development issues using the conservation of the charismatic and endangered snow leopard as a
flagship. This iconic and culturally treasured great cat is a good indicator species as it quickly reacts to
habitat disturbance and its successful conservation requires sustainable longterm systemic solutions to the
threats impacting the quality of habitats.
• The foundation of the GSLEP is 12 individual National Snow Leopard and Ecosystems Priorities
(NSLEPs). After a process of sharing knowledge and known good practices and developing a common
vision, the NSLEPs were developed to incorporate a set of priority, concrete project activities to be
implemented to meet national goals and, collectively, the overarching global goal.
• The 12 Snow Leopard Range Countries according to GSLEP are:
o India
o Mongolia
o Russia
o China
o Bhutan
o Afghanistan
o Kazakhstan
o Kyrgystan
o Nepal
o Pakistan
o Tajikistan
o Uzbekistan

Q 38.C
• Coal is the most important and abundant fossil fuel in India. It accounts for 55% of the country's
energy needs. Hard coal deposit spread over 27 major coalfields are mainly confined to eastern and
south-central parts of the country. A cumulative total of 2,93,497 million tonnes of geological resources of
Coal up to a depth of 1200 meters have so far been estimated in the country. Hence statement 1 is
correct.
• The coal resources of India are available in older Gondwana (570 million years to 245 million years
ago) formations of peninsular India and younger tertiary (60 to 15 million years ago) formations of
north-eastern region.
• Gondwana coal belongs to the carboniferous period. It is found in the Damodar, Mahanadi, Godavari,
and Narmada valleys. Raniganj, Jharia, Bokaro, Ramgarh, Giridih, Chandrapur, Karanpura, Tatapani,
16 www.visionias.in ©Vision IAS

Google it:- https://upscpdf.com


https://t.me/UPSC_PDF Download From > https://upscpdf.com https://t.me/UPSC_PDF

Talcher, Himgiri, Korba, Penchgati, Sarguja, Kamthi, Wardha valley, Singreni (A.P.) and Singrauli are
some of the important coal mines of the Gondwana formations. The Jharguda coal mine (Chhattisgarh) is
the thickest coal seam 132 meters of the Gondwana period, followed by the Kargali seam near Bokaro
belong to the Gondwana period.
• Types of Coal in India: Anthracite, Bituminous, and Lignite. They are classified on the basis of
carbon content. Anthracite coal consists of 80-98% of carbon. Bituminous coal consists of 60-80%
of carbon content. Lignite consists only of 20-60% of carbon content.
• Anthracite is a hard, compact variety of mineral coal that has a high luster. It has the highest carbon
content, the fewest impurities, and the highest calorific content of all types of coal. The carbon content is
between 92.1% and 98%. It is used mainly in power generation, in the metallurgy sector. Anthracite
accounts for about 1% of global coal reserves and is mined in only a few countries around the world.
China accounts for the majority of global production; other producers are Russia, Ukraine, North Korea,
Vietnam, the UK, Australia, and the US. Anthracite (more than 80% carbon content) is the best quality of
coal. In India, it is found only in Jammu and Kashmir.
• Bituminous coal or black coal is relatively soft coal containing a tarlike substance called bitumen. It is of
higher quality than lignite coal but of poorer quality than anthracite. The carbon content of bituminous
coal is around 60-80%; the rest is composed of water, air, hydrogen, and sulfur. About 80 percent of
the coal deposits in India are of a bituminous type and is of non-coking grade. Hence statement 2 is
correct.
• The lignite reserves stand at a level of 41.96 billion tones, of which 90% occur in the southern State of
Tamil Nadu. Other states where lignite deposits have been located are Rajasthan, Gujarat, Kerala, Jammu
& Kashmir, and a union territory of Puducherry.

Q 39.A
• The tropical climate is favourable for the cultivation of a wide range of cash crops along with lumbering
in tropical forests.
• Indonesia is the largest producer of palm oil, followed by Malaysia - both countries account for 84% of
the worlds palm production. There has recently been an increase in palm oil production in South America
via Colombia, Ecuador and Guatemala.
• Jute is confined almost entirely to the Ganges - Brahmaputra delta, in India and Bangladesh. It is hard
fibre and is mostly used for the manufacture of sacks. Jute crop requires humid climate with temperature
fluctuating between 24 degree Celsius and 38 degree Celsius. The minimum rainfall required for jute
cultivation is 1000 mm. New grey alluvial soil of good depth receiving silt from annual floods is most
suitable for jute growth. India is the world's largest producer of raw jute and jute goods, contributing
to over 50 percent and 40 percent respectively of global production.
• The tropical forests mainly consist of tropical deciduous trees, of which teak is very important. It is
valuable on account of its great durability, strength, immunity to shrinkage, fungus attack, and
insects. Myanmar is the leading producer of teak in the world. In India, it is mostly grown in the
peninsular states.
• Hence only pair 1 is correctly matched.

Q 40.B
• Statement 1 is not correct: Migration may be permanent, temporary or seasonal. It may take place
from rural to rural areas, rural to urban areas, urban to urban areas and urban to rural areas.
Immigration: Migrants who move into a new place are called Immigrants. Emigration: Migrants who
move out of a place are called Emigrants.
• In the Census of India migration is enumerated on two bases : (i) place of birth, if the place of birth is
different from the place of enumeration (known as life-time migrant);(ii) place of residence, if the place of
the last residence is different from the place of enumeration (known as migrant by place of last
residence).
• Statement 2 is correct: The distribution of male and female migrants in different streams of intra-state
and inter-state migration is presented in the adjoining figures. It is clearly evident that females
predominate the streams of short distance rural to rural migration in both types of migration.
Contrary to this, men predominate the rural to an urban stream of inter-state migration due to economic
reasons.

17 www.visionias.in ©Vision IAS

Google it:- https://upscpdf.com


https://t.me/UPSC_PDF Download From > https://upscpdf.com https://t.me/UPSC_PDF

Q 41.D
• Primary activities are directly dependent on the environment as these refer to utilisation of earth’s
resources such as land, water, vegetation, building materials and minerals.
• It, thus includes hunting and gathering, pastoral activities, fishing, forestry, agriculture, and
mining, and quarrying.
• Hence all the options are correct.

Q 42.A
• With the process of colonization, many large plantations have been established in the equatorial regions of
the world.
• The hot and wet climate has proved to be very favourable for the cultivation of certain crops like rubber,
cocoa, tea, etc. that are highly valued in the industrial world. Hence option 1 is correct.
• High temperature and abundant rainfall support a luxuriant type of vegetation-the tropical rain forest.
These forests are spread throughout the equatorial region. In fact, these forests are so dense and widely
spread, that some regions like the Amazon are described as the 'lungs of the earth'.
• Despite having heavy leaf-fall and the decomposition of leaves by bacteria, the soil here is not very
nutrient-rich, as the torrential downpours soon wash out most of the soil nutrients and the soil deteriorates
rapidly. Hence option 2 is not correct.
• The equatorial conditions are ideal for the survival of germs and bacteria and are easily transmitted
through moist air. Insects and pests not only spread diseases but are injurious to crops. Eg. Coffee Berry
disease. Hence option 3 is not correct.

Q 43.D
• Recent Context: Styrene gas leak claimed 11 lives in Visakhapatnam in Andhra Pradesh. The
source of the leak was a styrene plant owned by South Korean electronics giant LG.
About Styrene Gas:
• Styrene is an organic compound with the formula C8H8, its molecular structure as pictured. It is
a derivative of benzene (C6H6). It is stored in factories as a liquid, but evaporates easily, and has to
be kept at temperatures under 20°C.
• It is a flammable substance that is used in the manufacturing of polystyrene plastics, fiberglass, rubber,
and latex. Hence statement 1 is correct.
• Styrene is also found in vehicle exhaust, cigarette smoke, and in natural foods like fruits and
vegetables. Hence statement 2 is correct.
• According to The Manufacture, Storage, and Import of Hazardous Chemicals Rules, 1989, styrene is
classified as a toxic and hazardous chemical.
• Short-term exposure to the substance can result in respiratory problems, irritation in the eyes, irritation in
the mucous membrane, and gastrointestinal issues. Hence statement 3 is correct.
• And long-term exposure could drastically affect the central nervous system and lead to other related
problems like peripheral neuropathy.
Q 44.C
• Recently, the National Rural Infrastructure Development Agency (NRIDA) has announced that coir
geotextiles will be used for the construction of rural roads under the Pradhan Mantri Gram Sadak Yojana
(PMGSY-III). Hence statement 2 is correct.
• NRIDA is an agency under the Ministry of Rural Development (not a statutory body). Hence statement 1
is correct.
18 www.visionias.in ©Vision IAS

Google it:- https://upscpdf.com


https://t.me/UPSC_PDF Download From > https://upscpdf.com https://t.me/UPSC_PDF

• The agency is associated with various Government programs related to rural development such as
PMGSY, MGNREGA, etc.
• PMGSY New Technology Guidelines: i
o The new guidelines encourage locally available materials and the use of green technologies for the
construction of roads under Pradhan Mantri Gram Sadak Yojana (PMGSY).
o The State Governments are required to propose a minimum of 15% of the total length of annual road
proposals under new technologies such as cement stabilization, Lime stabilization, cold mix, waste
plastics, cell filled concrete, paneled cement concrete pavement, fly ash, etc.
• Out of this, 5% of roads are to be constructed using Indian Road Congress (IRC) accredited technology.
The IRC has now accredited coir Geotextiles for the construction of rural roads. Thus, a 5% length of the
rural roads under PMGSY-III will be constructed using coir geotextiles.
Q 45.A
• Lignite Coal:
o Lignite coal is found in Arunachal Pradesh and West Bengal (Darjeeling District).
o The largest lignite deposits of the country are at Neyveli in the state of Tamil Nadu.
o Coal generally has a low carbon and a high percentage of moisture and sulfur, along with organic
material. Hence statement 2 is correct.
o It is almost 57-58 percent of the total production of tertiary coal and thus, it contributes to 1-
1.5% of total production. It is considered the lowest rank of coal due to its relatively low heat
content. Hence statement 3 is correct.
o It has a carbon content around 20-40% percent. Hence statement 1 is correct.
o It is mined all around the world, is used almost exclusively as a fuel for steam-electric power
generation, and is the coal that is most harmful to health.
Q 46.A
• Population aging is the process by which the share of the older population becomes proportionally larger.
This is a new phenomenon of the twentieth century. In most of the developed countries of the world, the
population in the higher age groups has increased due to increased life expectancy. With a reduction in
birth rates, the proportion of children in the population has declined. Hence only option 1 and option 2
are correct.
• Population ageing is an increasing median age in a population due to declining fertility rates and rising
life expectancy. Most countries have a rising life expectancy and an ageing population. Japan has the
highest proportion of the oldest population in the world as the median age of its population is around 47
years.
• Demographic dividend, as defined by the United Nations Population Fund (UNFPA) means, "the
economic growth potential that can result from shifts in a population’s age structure, mainly when the
share of the working-age population (15 to 64) is larger than the non-working-age share of the population
(14 and younger, and 65 and older). Therefore an increase in the proportion of the working population
does not contribute to population ageing. Hence option 3 is not correct.
Q 47.D
• The savanna is said to be the natural cattle country and many of the native people are, in fact, herdsmen or
pastoralists. The cattle provide the people with milk, blood, and meat.
• But, the native zebu cattle are bony and yield little meat or milk. They often fall victim to tropical
diseases like the ngana or sleeping sickness carried by the tsetse fly in Africa. Hence statements 1 and 2
are correct.
• In the African savanna, certain native herdsmen like the Masai treat cattle as prestige animals, not for
slaughtering, this pose difficulties towards the commercialization of the cattle industry. Hence statement
3 is correct.

Q 48.C
• Rivers as Inland waterways for domestic and international transport and trade has great significance.
Many rivers have been modified to enhance their navigability by dredging, stabilizing river banks, and
building dams and barrages for regulating the flow of water.
• The international waterways are governed by various United Nations agreements like the 'European
Agreement on Main Inland Waterways of International Importance (AGN)'
• The following river waterways are some of the world's important highways of commerce:
o The Rhine Waterways: The Rhine river flows through Netherlands and Germany and is navigable
for 700 km from Rotterdam, in the Netherlands to Basel in Switzerland. It connects the industrial
areas of Switzerland, Germany, France, Belgium, and the Netherlands with the North Atlantic Sea
Route. Hence, pair 1 is not correctly matched.
19 www.visionias.in ©Vision IAS

Google it:- https://upscpdf.com


https://t.me/UPSC_PDF Download From > https://upscpdf.com https://t.me/UPSC_PDF

o The Danube Waterway: The Danube river rises in the Black Forest of Germany and flows
eastwards through many countries. It is navigable up to Taurna Severin, Romania. Hence pair 2 is
not correctly matched.
o The Volga Waterway: It starts in Russia, it provides a navigable waterway of 11,200 km and drains
into the Caspian Sea. The Volga-Moscow canal connects it with the Moscow region and the Volga-
Don Canal with the Black Sea. Hence pair 3 is correctly matched.

Q 49.A
• UN Children’s Fund (UNICEF) has published a report named UN the “Lost at Home” report. Hence
option (a) is the correct answer.
• Almost 33 million new displacements were recorded in 2019 — around 25 million were due to natural
disasters and 8.5 million as a consequence of conflict and violence.
• There were 12 million new displacements of children in 2019: around 3.8 million of them caused
by conflict and violence, and 8.2 million, due to disasters linked mostly to weather-related events.
• Natural disasters resulted in more new displacements than conflict and violence.
• Almost 10 million new displacements in 2019 were recorded in East Asia and the Pacific (39 %) —
and almost the same number in South Asia (9.5 million).
• Coronavirus intensifying suffering: Camps or informal settlements are often overcrowded, and lack
adequate hygiene and health services. Physical distancing is often not possible, creating conditions that
are highly conducive to the spread of disease.
• Risks internally displaced children face include child labour, child marriage, trafficking.
• The largest number of internally displaced children due to conflict are found in the Middle East and North
Africa (MENA) and sub-Saharan Africa. Internally displaced persons are concentrated in two regions —
the Middle East and North Africa and West and Central Africa.

Q 50.A
• Road transport is the most economical for short distances. The world's total motorable road length is only
about 15 million km, of which North America accounts for 33 percent. The highest road density and the
highest number of vehicles are registered in this continent. Hence statement 1 is correct.
• Railways are a mode of land transport for bulky goods and passengers over long distances. Europe has
one of the densest rail networks in the world. Belgium has the highest density of 1 km of railways for
every 6.5 sq km area.
• Rivers, canals, lakes, and coastal areas are important waterways. The development of inland waterways is
dependent on the navigability width and depth of the channel, continuity in the water flow, and transport
technology in use. Asia with major rivers like the Ganga-Brahmaputra river system, Beijing-Hangzhou
Grand Canal, etc. has the highest length of inland waterways. Hence statement 2 is not correct.

Q 51.A
• Statement 1 is correct: As per census 2011, there are very large Urban Areas with more than 10
million persons in the country. These are known as Megacities. These are Greater Mumbai UA, Delhi
UA, and Kolkata UA. The growth in population in the Mega Cities has slowed down considerably during
the last decade. Greater Mumbai UA, which had witnessed 30.47% growth in population during 1991-
2001 has recorded 12.05% during 2001-2011. Similarly, Delhi UA (from 52.24% to 26.69% in 2001-
2011) and Kolkata UA (from 19.60% to 6.87% in 2001-2011) have also slowed down considerably.
• Statement 2 is not correct: The level of urbanization is measured in terms of the percentage of urban
population to the total population. The level of urbanization in India in 2001 was 28 percent. The total
urban population in the country as per Census 2011 is more than 377 million constituting 31.16% of the
total population.

Q 52.A
• HVJ an abbreviation for Hazira-Vijaipur-Jagdishpur (also known as HBJ where B stands for
Bijeypur, another name of Vijaipur) is India's first cross-state gas pipeline. The project was started in
1986 after the incorporation of GAIL (India) Limited to supply gas to the fertilizer plants located in the
state of Uttar Pradesh. It passed through Gujarat, Madhya Pradesh, and Uttar Pradesh. The first
phase of the project consisting of a non-branched 1,750-kilometer grid was commissioned in 1997. Later
on, the system was expanded with additional branches to supply gas for industrial and domestic use
in the states of Rajasthan, Haryana, and NCT, which increased the total grid length to 3,474 km.
• In the year 1998, newly established Indraprastha Gas Limited took control of the Delhi branch of the
pipeline to set up a citywide gas grid.
20 www.visionias.in ©Vision IAS

Google it:- https://upscpdf.com


https://t.me/UPSC_PDF Download From > https://upscpdf.com https://t.me/UPSC_PDF

Q 53.A
• Coal is one of the important minerals which is mainly used in the generation of thermal power and
smelting of iron ore. It is one of the most mined minerals from the earth. According to one estimate,
proven coal reserves are 860, 938 million tonnes.
• Of the three fossil fuels (petroleum, natural gas, and coal), coal has the most widely distributed
reserves; coal is mined in over 100 countries, and on all continents except Antarctica. The largest proved
reserves are found in the United States, Russia, China, Australia, and India. Coal is found majorly
in forms of Lignite and Anthracite. Hence statement 1 is correct.
• In terms of production, China is the top coal producer since 1983. In 2011 China produced 3,520
million tonnes (mt) of coal – 49.5% of 7,695 million tonnes world coal production. In 2011 other large
producers were United States (993 mt), India (589 mt), European Union (576 mt), and Australia (416
mt). Hence statement 2 is not correct.
• Top coal exporting countries are Australia with 27% and Indonesia with 26% of total world coal export in
2010. Japan is the largest coal importer with 17% of total world coal import seconded by China having a
share of 16% in 2010.

Q 54.C
• Recent context: An invasive mussel native to the South and Central American coasts is spreading
quickly in the backwaters of Kerala. Hence option (c) is correct.
• The rapid spread of the Charru mussel (Mytella strigata) may have been triggered by Cyclone Ockhi
which struck the region in 2017.
• With a population as high as 11,384 per sq metre here, it has replaced the Asian green mussel (Perna
Viridis) and the edible oyster Magallana bilineata (known locally as muringa).
• Invasive Species:
o It is an organism that causes ecological or economic harm in a new environment where it is not native.

Q 55.B
• Statement 1 is not correct: A population pyramid, also called an "age-gender-pyramid", is a graphical
illustration that shows the distribution of various age groups in a population (typically that of a country or
region of the world), which forms the shape of a pyramid when the population is growing.

• Statement 2 is correct: Expanding Populations: The age-sex pyramid of Nigeria is a triangular-shaped


pyramid with a wide base and is typical of less developed countries. These have larger populations in
lower age groups due to high birth rates. If you construct the pyramids for Bangladesh and Mexico, it
would look the same.

21 www.visionias.in ©Vision IAS

Google it:- https://upscpdf.com


https://t.me/UPSC_PDF Download From > https://upscpdf.com https://t.me/UPSC_PDF

• Statement 3 is correct: Australia’s age-sex pyramid is bell-shaped and tapered towards the top. This
shows birth and death rates are almost equal leading to a near-constant population

Q 56.A
• Coffee is a tropical plantation crop. Its seeds are roasted, ground and are used for preparing a beverage.
There are three varieties of coffee i.e. arabica, robusta and Liberia. India mostly grows superior quality
coffee, arabica, which is in great demand in the international market. Hence statement 2 is correct.
• But India produces only about 3.7 percent coffee in the world and ranks seventh after Brazil, Vietnam,
Colombia, Indonesia, Ethiopia, and Honduras in 2016. Hence statement 1 is not correct.
• Coffee is cultivated in the highlands of Western Ghats in Karnataka, Kerala and Tamil Nadu.
• Karnataka alone accounts for more than two-thirds of the total production of coffee in the country. Hence
statement 3 is not correct.

Q 57.C
• Statement 1 is correct: Crude oil may be referred to as sweet if it contains relatively little sulfur
(0.5%) or sour if it contains substantial amounts of sulfur. Sweet crude requires less energy to be
extracted and once extracted, yields higher quality gasoline as well as larger quantities of it.
• Statement 2 is correct: Iraq is one of the leading producers of sweet crude. Major locations where
sweet crude is found include the Appalachian Basin in Eastern North America, Western Texas, the
Bakken Formation of North Dakota and Saskatchewan, the North Sea of Europe, North Africa, Australia,
and the Far East including Indonesia. Sour crude, on the other hand, has a high level of impurities in it,
namely sulfur, which must first be removed before being processed into gas and other petroleum-based
products. Venezuela is a leading producer of sour crude oil. Sour crude is more common in the Gulf of
Mexico, Mexico, South America, and Canada. Crude produced by OPEC Member Nations also tends to
be relatively sour, with an average sulfur content of 1.77%.
• Iraq has maintained its position as the largest crude oil supplier to India in 2018-2019, supplying
more than 46.61 Million Tonne (MT) of crude oil last financial year ended March 2019, official data
sourced from Directorate General of Commercial Intelligence and Statistics (DGCIS) showed.

Q 58.C
• Footloose industry is a general term for an industry that can be placed and located at any location without
effect from factors such as resources or transport. Hence statement 1 is correct. Diamonds and computer
chips are some examples of footloose industries.
• The reason for the growth of footloose industries lies in the rapid development of highly sophisticated
products requiring a great deal of scientific research and development.
• Footloose industries enable quick product improvement of their products to suit the market demand.
• Characteristic features of footloose Industries which favor the free choice of location:
o light industries that often do not use raw materials but component parts

22 www.visionias.in ©Vision IAS

Google it:- https://upscpdf.com


https://t.me/UPSC_PDF Download From > https://upscpdf.com https://t.me/UPSC_PDF

o power requirements, usually only electricity — available from the national grid
o the end product is small and often cheaper and easier to move.
o employs a small labor force
o non-polluting industries that can be located near residential areas. Hence statement 2 is correct.
o accessibility - needs to be near a road network
Q 59.B
• People migrate for better economic and social life. There are two sets of factors that influence migration.
• The Push factors make the place of origin seem less attractive for reasons like unemployment, poor
living conditions, political turmoil, unpleasant climate, natural disasters, epidemics, and socio-
economic backwardness. Epidemics in urban areas is, therefore, a Push factor. Hence option 2 is
correct.
• The Pull factors make the place of destination seem more attractive than the place of origin for reasons
like better job opportunities and living conditions, peace and stability, the security of life and property,
and pleasant climate. Therefore better-living conditions in rural areas is a 'Pull factor'. Hence option 1 is
not correct.
• Lack of employment opportunities in rural areas is a 'push factor' for rural to urban migration but not for
the mass exodus of migrant laborers from urban to rural areas. Hence option 3 is not correct.
Q 60.C
• There are three distinct crop seasons in the northern and interior parts of the country, namely Kharif,
Rabi, and Zaid.
• The Kharif season largely coincides with Southwest Monsoon under which the cultivation of
tropical crops, such as rice, cotton, jute, jowar, bajra, and tur is possible. Hence pair 1 is not
correctly matched.
• The rabi season begins with the onset of winter in October-November and ends in March-April. The
low-temperature conditions during this season facilitate the cultivation of temperate and
subtropical crops such as wheat, gram, and mustard. Hence pair 2 is not correctly matched.
• Zaid is a short duration summer cropping season beginning after harvesting of rabi crops. The
cultivation of watermelons, cucumbers, vegetables and fodder crops during this season is done on
irrigated lands. Hence pair 3 is correctly matched.
• However, this type of distinction in the cropping season does not exist in the southern parts of the
country. Here, the temperature is high enough to grow tropical crops during any period in the year
provided the soil moisture is available. Therefore, in this region same crops can be grown thrice in an
agricultural year provided there is sufficient soil moisture.
Q 61.D
• Lumbering is defined as the felling of economic trees in the forest, which can be used in domestic,
industrial, or commercial purposes.
• Factors that favor lumbering :
o Limited species forests such as coniiferous forests.
o Lumbering is normally carried out in the winter when the sap ceases to flow. This makes felling
much simpler. Hence, statement 1 is not correct.
o The snow-covered ground makes logging and haulage [commercial transport of goods] a relatively
easy job.
o The logs are dragged to the rivers and float to the saw-mills downstream when the rivers thaw
[unfreeze] in spring. This has greatly assisted the lumbering industry in eastern Canada and Sweden.
o Cheap hydro-electricity for driving the saw-mills is harnessed in the mountainous uplands of
North America and Europe and has greatly assisted the lumbering industry.
• Factors hindering Lumbering :
o High species diversity as seen in Equatorial and Tropical forests.
o The heaviness of the tropical hardwoods makes them unbale to float in rivers, thus requiring the
development of various modes of transport in the forests. Hence, statement 2 is correct.
• It is the most important occupation of the Siberian type of climate. Hence, statement 3 is correct.
Q 62.D
• The equatorial regions are generally sparsely populated, inhabited by primitive people that live as hunters
and collectors and the more advanced ones practice shifting cultivation.
• In the Amazon basin the Indian tribes collect wild rubber, in the Congo basin the Pygmies gather nuts
and in the jungles of Malaysia the Orang Asli make all sorts of cane products and sell them to people in
villages and towns. Hence, only pair 2 is correctly matched.
• With the cultivation of plantation crops like rubber, cocoa, sugar, coffee, tea, etc.many tribes have begun
to lead a settled life.
23 www.visionias.in ©Vision IAS

Google it:- https://upscpdf.com


https://t.me/UPSC_PDF Download From > https://upscpdf.com https://t.me/UPSC_PDF

• The recent Amazon fires have destroyed the habitat of the uncontacted Indian tribes and they are now at
risk as some of their last forest refuges are being consumed by flames.
Q 63.C
• Phase I: The period from 1901-1921 is referred to as a period of stagnant or stationary phase of growth
of India’s population, since in this period growth rate was very low, even recording a negative growth rate
during 1911-1921. Both the birth rate and death rate were high keeping the rate of increase low
(Appendix–iii). Poor health and medical services, illiteracy of people at large and inefficient
distribution system of food and other basic necessities were largely responsible for a high birth and death
rates in this period.
• Phase II: The decades 1921-1951 are referred to as the period of steady population growth. An
overall improvement in health and sanitation throughout the country brought down the mortality rate.
At the same time better transport and communication system improved distribution system. The crude
birth rate remained high in this period leading to higher growth rate than the previous phase. This
is impressive at the backdrop of Great Economic Depression, 1920s and World War II.
• Phase III: The decades 1951-1981 are referred to as the period of population explosion in India,
which was caused by a rapid fall in the mortality rate but a high fertility rate of population in
the country. The average annual growth rate was as high as 2.2 per cent. It is in this period, after
the Independence, that developmental activity were introduced through a centralised planning process
and economy started showing up ensuring the improvement of living condition of people at
large. Consequently, there was a high natural increase and higher growth rate. Besides, increased
international migration bringing in Tibetans, Bangladeshis, Nepalies and even people from Pakistan
contributed to the high growth rate. Hence option (c) is the correct answer.
• Phase IV: In the post 1981 till present, the growth rate of country’s population though remained high, has
started slowing down gradually (Table 1.1). A downward trend of crude birth rate is held responsible for
such a population growth. This was, in turn, affected by an increase in the mean age at marriage,
improved quality of life particularly education of females in the country.
Q 64.B
• Human geography is the study of the inter-relationship between the physical environment and socio-
cultural environment created by human beings through mutual interaction with each other.
• The following are the different schools of Human geography:
o Possibilism: Nature provides humans with a range of possibilities and human beings depending upon
the level of technology and development can choose among these alternatives. Thus it is the human
beings who are deciding their own fate as nature is neutral. Its proponents were Vidal de la Blache,
Lucian Febre, etc.
o Environmental Determinism: It is a school of thought which believes that all the activities of
humans are controlled by nature. The primitive humans were greatly influenced by their
interactions with the natural environment. The level of technology was very low and humans
listened to Nature, was afraid of its fury and worshipped it. Proponents of this theory were Ratzel,
Ellen Semple, etc. Hence, Option B is correct.
o Cultural Determinism: It talks about the role of culture in which we are raised, determines the future
human activities and state of a nation. Example: Marx's economic theories where the state of economy
determines the behaviour and activities of humans.
o Neo-Determinism: Introduced by Griffith Taylor, also called as 'stop and go determinism'. According
to him humans are like traffic police who can stop traffic for some time but he would have to allow it
to ultimately let go. Similarly humans can alter the environment upto certain extent but ultimately it
would be nature which would have the last say.
Q 65.B
• India is rich in good quality iron ores.
o Magnetite is the finest iron ore with a very high content of iron up to 70 per cent. It has excellent
magnetic qualities, especially valuable in the electrical industry.
o Hematite ore is the most important industrial iron ore in terms of the quantity used, but has a slightly
lower iron content than magnetite. (50-60 per cent).
• The major iron ore belts in India are:
o Odisha-Jharkhand belt: In Odisha high grade hematite ore is found in Badampahar mines in
the Mayurbhanj and Kendujhar districts. In the adjoining Singbhum district of Jharkhand haematite
iron ore is mined in Gua and Noamundi.
o Durg-Bastar-Chandrapur belt lies in Chhattisgarh and Maharashtra. Very high grade hematites
are found in the famous Bailadila range of hills in the Bastar district of Chhattisgarh. The range
of hills comprise of 14 deposits of super high grade hematite iron ore. It has the best physical
24 www.visionias.in ©Vision IAS

Google it:- https://upscpdf.com


https://t.me/UPSC_PDF Download From > https://upscpdf.com https://t.me/UPSC_PDF

properties needed for steel making. Iron ore from these mines is exported to Japan and South Korea
via Vishakhapatnam port.
o Ballari-Chitradurga-Chikkamagaluru-Tumakuru belt in Karnataka has large reserves of iron
ore. The Kudremukh mines located in the Western Ghats of Karnataka are a 100 per cent
export unit. Kudremukh deposits are known to be one of the largest in the world. The ore is
transported as slurry through a pipeline to a port near Mangaluru.
• Hence option (b) is the correct answer.

Q 66.C
• The Special 301 Report identifies trading partners that do not adequately or effectively protect and
enforce Intellectual Property (IP) rights or otherwise deny market access to U.S. innovators and
creators that rely on the protection of their IP rights.
• The report is released annually by the United States Trade Representative (USTR).
• Trading partners that currently present the most significant concerns regarding IP rights are placed on
the Priority Watch List or Watch List.
• USTR identified 33 countries for these lists in the Special 301 Report:
o Algeria, Argentina, Chile, China, India, Indonesia, Russia, Saudi Arabia, Ukraine, and Venezuela are
on the Priority Watch List.
o Brazil, Canada, Colombia, Egypt, Kuwait, Lebanon, Mexico, Pakistan, Thailand, Turkey,
Turkmenistan, the United Arab Emirates, Uzbekistan and Vietnam are on the Watch List.
• Hence option (c) is the correct answer.

Q 67.C
• The population growth or population change refers to the change in number of inhabitants of a territory
during a specific period of time. This change may be positive as well as negative. It can be expressed
either in terms of absolute numbers or in terms of percentage. Population change in an area is an
important indicator of economic development, social upliftment and historical and cultural background of
the region.
• Change of population in particular area between two points of time is known as growth of population. For
example, if we deduct the population of India 2001 (102.70 crore) from population of 2011 (121.02 crore)
then we shall get the growth of population (18.15 crores) in actual numbers.
• Growth Rate of Population : This is the change of population expressed in percentage. Natural Growth
of Population: This is the population increased by difference between births and deaths in a particular
region between two points of time.
o Natural Growth = Births – Deaths
o Actual Growth of Population: This is Births – Deaths + In Migration – Out Migration. Hence
option (c) is the correct answer.

Q 68.A
• Public Utility Services are those business undertakings engaged in supplying essential goods and/or
services of daily necessity for the general public. Hence, statement 1 is correct.
• Section 2 (n) of Industrial Disputes Act empowers Government to declare any industry specified in
first Schedule of IDA to be a ‘public utility service’ for a period of six months by issuing a
notification, if it satisfies that public emergency or public interest so requires. Hence, statement 2 is
not correct.
• Ministry of Labour and Employment through a notification has declared ‘banking industry’ as a ‘public
utility service’ for six months (till October 21) under Industrial Disputes Act, 1947 (IDA).
• IDA also defines following industries as ‘public utility services’
o Railway service (or any transport service for carriage of passengers or goods by air)
o Service in connection with working of, any major port or dock;
o Section of an industrial establishment, on working of which safety of establishment or workmen
employed therein depends;
o Postal, telegraph or telephone service
o Industry which supplies power, light or water to public
o System of public conservancy or sanitation
• Impact of declaring an industry as public utility service: Section 22(1) of IDA provides that no person
employed in public utility service shall go on strike in breach of contract.
o Without giving to employer notice of strike within six weeks before striking.
o Within fourteen days of giving such notice; or
25 www.visionias.in ©Vision IAS

Google it:- https://upscpdf.com


https://t.me/UPSC_PDF Download From > https://upscpdf.com https://t.me/UPSC_PDF

o Before the expiry of the date of strike specified in any such notice as aforesaid
o During the pendency of any conciliation proceedings before a conciliation officer and seven days after
the conclusion of such proceedings.
• Similar conditions apply for ‘lockout’ of public utility services.

Q 69.B
• The Tropical Monsoon and Tropical Marine Climates are found in the zones between 5° and 30° latitudes
on either side of the equator.
• Tropical Monsoon climate is best developed in the Indian sub-continent, Burma, Thailand, Laos,
Cambodia, parts of Vietnam and south China and northern Australia, which experience the reversal of
winds.
• Tropical Marine Climate, is experienced in Central America, West Indies, north-eastern Australia, the
Philippines, parts of East Africa, Madagascar, the Guinea Coast, and eastern Brazil, which are under the
influence of the on-shore Trade Winds all the year-round.
• Shifting cultivation, the most primitive form of farming is widely practiced, in the interiors of these
areas till the date
• In this instead of rotating the crops in the same field to preserve fertility, the tribesmen move to a new
clearing when their first field is exhausted.
• Maize, dry padi, sweet potatoes and some beans are the most common crops.
• Farming is entirely for subsistence.
• Shifting cultivation is so widely practiced amongst indigenous peoples that different local names are
used in different countries.
Region Local Name
Malaysia Ladang
Sri Lanka Chena
Myanmar Taungiya
Thailand Tamrai
Java Humah
Philippines Caingin
Africa & Central America Milpa
North East India Jhum
Q 70.C
• Both statements 1 and 2 are correct: The Regional Comprehensive Economic Partnership (RCEP) is
a proposed agreement between the member states of the Association of Southeast Asian Nations
(ASEAN) and its free trade agreement (FTA) partners.
• The pact aims to cover trade in goods and services, intellectual property, etc. The negotiations are
focused on the following: Trade in goods and services, investment, intellectual property, dispute
settlement, e-commerce, small and medium enterprises, and economic cooperation. It was introduced
during the 19th Asean meet held in November 2011.
• Member states of Asean and their FTA partners are Brunei, Cambodia, Indonesia, Laos, Malaysia,
Myanmar, the Philippines, Singapore, Thailand, Vietnam, China, Japan, India, South Korea, Australia and
New Zealand.
• However India, as of now (July 2020), has decided against joining the 16-nation Regional Comprehensive
Economic Partnership (RCEP) trade deal, saying it was not shying away from opening up to global
competition across sectors, but it had made a strong case for an outcome which would be favourable to all
countries and all sectors.

Q 71.D
• Some of the factors influencing industrial locations are as under:
o Access to Market: The existence of a market for manufactured goods is the most important factor in
the location of industries. Remote areas inhabited by a few people offer small markets. The developed
regions of Europe, North America, Japan and Australia provide large global markets as the purchasing
power of the people is very high.
o Access to Raw Material: Raw material used by industries should be cheap and easy to transport.
Industries based on cheap, bulky and weight-losing material (ores) are located close to the sources of
raw material such as steel, sugar, and cement industries.
26 www.visionias.in ©Vision IAS

Google it:- https://upscpdf.com


https://t.me/UPSC_PDF Download From > https://upscpdf.com https://t.me/UPSC_PDF

o Access to Labour Supply: Labour supply is an important factor in the location of industries.
o Access to Sources of Energy: Industries which use more power are located close to the source of the
energy supply such as the aluminium industry.
o Access to Transportation and Communication Facilities: Speedy and efficient transport facilities
to carry raw materials to the factory and to move finished goods to the market are essential for the
development of industries. The cost of transport plays an important role in the location of industrial
units. Communication is also an important need for industries for the exchange and management of
information.
o Government Policy: Governments adopt ‘regional policies’ to promote ‘balanced’ economic
development and hence set up industries in particular areas.
o Access to Agglomeration Economies/ Links between Industries: Many industries benefit from
nearness to a leader-industry and other industries. These benefits are termed as agglomeration
economies.
• Hence option (d) is the correct answer.

Q 72.B
• Though most of the states in India have only one state capital, however Maharashtra (Mumbai and
Nagpur) and Himachal Pradesh (Shimla and Dharmshala) have two capitals each and Andhra Pradesh has
three capitals (Amaravati - Legislative capital, Visakhapatnam - Executive capital and Kurnool - Judicial
capital).
• Among the given cities the correct order in North to South direction is Raipur-Vishakhapatnam-
Bengaluru-Thiruvananthapuram. Hence, the correct answer is option (b).

Q 73.A
• Both growth and development refer to changes over a period of time. The difference is that growth is
quantitative and value-neutral. It may have a positive or a negative sign. This means that the change
may be either positive (showing an increase) or negative (indicating a decrease). Development means a
qualitative change which is always value positive. Hence statement 1 and statement 2 are correct.

27 www.visionias.in ©Vision IAS

Google it:- https://upscpdf.com


https://t.me/UPSC_PDF Download From > https://upscpdf.com https://t.me/UPSC_PDF

• This means that development cannot take place unless there is an increment or addition to the
existing conditions. Development occurs when positive growth takes place. Yet, positive growth
does not always lead to development. Hence statement 3 is not correct. Development occurs when
there is a positive change in quality.
• For example, if the population of a city grows from one lakh to two lakhs over a period of time, we say
the city has grown. However, if a facilities like housing, provision of basic services and other
characteristics remain the same, then this growth has not been accompanied by development. The concept
of human development was introduced by Dr Mahbub-ul-Haq. Dr Haq has described human development
as development that enlarges people’s choices and improves their lives.

Q 74.D
• The estimated reserves of Natural Gas in India is 1339.57 Billion Cubic Meters (BCM) in March 2018.
Majority of the estimated reserves of Natural Gas is found on the Eastern Offshore fields (39%) in the
Krishna-Godavari basins.
• The following table shows the estimated natural gas reserves in India by state/region March 2017:
Region Natural gas reserves(in BCM) Share of gas (%)
Arunachal Pradesh 0.93 0.07
Andhra Pradesh (Krishna-
48.31 3.75
Godavari Onshore Fields)
Assam 158.57 12.29
Coal Bed Methane 106.58 8.26
Eastern Offshore 507.76 39.37
Gujarat 62.28 4.83
Nagaland 0.09 0.01
Rajasthan (Mangala Area) 34.86 2.70
Tamil Nadu 31.98 2.48
Tripura (Moungram) 36.10 2.80
Western Offshore (Mumbai High
302.35 23.44
and Offshore of Gujarat)
Total 1,289.81 100
• Hence, the correct answer is option (d).

Q 75.D
• Majority of metropolitan and mega cities are urban agglomerations. An urban agglomeration may consist
of any one of the following three combinations: (i) a town and its adjoining urban outgrowths, (ii) two
or more contiguous towns with or without their outgrowths, and (iii) a city and one or
more adjoining towns with their outgrowths together forming a contiguous spread. Hence option
(d) is the correct answer.
• Examples of urban outgrowth are railway colonies, university campus, port area, military cantonment, etc.
located within the revenue limits of a village or villages contiguous to the town or city. According to
census 2011, an urban agglomeration is a continuous urban spread constituting a town and its adjoining
outgrowths (OGs), or two or more physically contiguous towns together with or without outgrowths
of such towns.

Q 76.C
• The population of India is larger than the total population of North America, South America and
Australia put together. Density of population, is expressed as number of persons per unit area. It helps in
getting a better understanding of the spatial distribution of population in relation to land. The density
of population in India (2011) is 382 persons per sq km. There has been a steady increase of more than
200 persons per sq km over the last 50 years as the density of population increased from 117
persons/ sq km in 1951 to 382 persons/sq km in 2011. The spatial variation of population densities in
the country ranges from as low as 17 persons per sq km in Arunachal Pradesh to 11,320 persons in the
National Capital Territory of Delhi.

28 www.visionias.in ©Vision IAS

Google it:- https://upscpdf.com


https://t.me/UPSC_PDF Download From > https://upscpdf.com https://t.me/UPSC_PDF

• Among the northern Indian States, Bihar (1106), West Bengal (1028) and and Uttar Pradesh (829)
have higher densities, while Kerala (860) and Tamil Nadu (555) have higher densities among the
peninsular Indian states. States like Assam, Gujarat, Andhra Pradesh, Haryana, Jharkhand, Odisha have
moderate densities. Hence option (c) is the correct answer.

Q 77.C
• The temperate grasslands are ideal for extensive wheat cultivation. Here winter wheat is grown and in
the months of spring, the climate is cool and moist that helps in the early growth of wheat. Hence,
statement 1 is correct.
• The light showers in the ripening period i.e at the beginning of summer months from convectional
sources, help to swell the grains to ensure a good yield.
• The warm and sunny summer is advantageous for harvesting and also enables the straw to be dried for
farm use. Hence, statement 2 is correct.
• In addition to this, the levelness of the Steppes and other temperate grasslands all over the world along
with the widespread black earth soil makes mechanized ploughing and harvesting a comparatively easy
job.

Q 78.A
• Sex ratio is defined in the Census of India as the number of females per 1,000 males in the
population. In the Population Census of 2011 it was revealed that the population ratio in India 2011
is 940 females per 1000 of males. Sex Ratio 2011 shows an upward trend from the census 2001 data.
Census 2001 revealed that there were 933 females to that of 1000 males.
• On an average, the world population reflects a sex ratio of 102 males per 100 females. The highest
sex ratio in the world has been recorded in Latvia where there are 85 males per 100 females. In contrast,
in Qatar, there are 311 males per 100 females. The sex ratio is favourable for females in 139 countries of
the world and unfavourable for them in the remaining 72 countries listed by the United Nations.
• In general, Asia has a low sex ratio. Countries like China, India, Saudi Arabia, Pakistan, Afghanistan have
a lower sex ratio. On the other extreme is a greater part of Europe (including Russia) where males are
in minority. A deficit of males in the populations of many European countries is attributed to a better
status of women, and an excessively male-dominated out-migration to different parts of the world in the
past.
• Hence the correct answer is option (a).

Q 79.C
• The Global Forest Resource Assessment is released by the United Nations Food and Agriculture
Organization (FAO).
• The FRA 2020 is based on the assessment of more than 60 forest-related variables in 236 countries and
territories in the period of 1990–2020.
• According to this, the rate of forest loss has declined in the period of 1990-2020.
• Total forest area:
o The world’s total forest area is 4.06 billion hectares (bha), which is 31% of the total land area.
o This area is equivalent to 0.52 hectares per person.
• Top countries in forest cover — the Russian Federation, Brazil, Canada, the United States of
America and China constituted more than 54% of the world’s forests.
• Forest loss: According to the report, the world has lost 178 million hectares (mha) of forest since 1990,
an area the size of Libya.
• Decline in rate of forest loss:
o The rate of net forest loss decreased substantially during the period of 1990–2020.
o It was 7.8 mha per year in the decade 1990–2000, 5.2 mha per year in 2000–2010 and 4.7 mha per
year in 2010–2020.
o This is due to a reduction in deforestation in some countries, plus increases in forest area in
others through afforestation and the natural expansion of forests.
• Areas that witnessed forest loss: Africa had the largest annual rate of net forest loss in 2010–2020, at
3.9 mha, followed by South America, at 2.6 mha.
• Areas that witnessed forest gain:
o Asia had the highest net gain of forest area in 2010–2020, followed by Oceania and Europe.
o However, Oceania experienced net losses of forest area in the decades 1990–2000 and 2000–2010.
• Types of forest loss: The largest proportion of the world’s forests are tropical (45%), followed by
boreal, temperate and subtropical.
29 www.visionias.in ©Vision IAS

Google it:- https://upscpdf.com


https://t.me/UPSC_PDF Download From > https://upscpdf.com https://t.me/UPSC_PDF

• Naturally regenerating forest areas worldwide decreased since 1990, but the area of planted forests has
increased.
• Plantation forest cover is 131 mha, about 3% of the global forest area.
o The highest percent of plantation forests are in South America while the lowest are in Europe.
• Protected forest areas worldwide estimate around 726 mha.
o South America has the highest share of forests in protected areas, at 31%.
o The protected forest areas increased by 191 mha since 1990.
• Hence option (c) is the correct answer.

Q 80.C
• Nuclear energy has emerged as a viable source in recent times. Important minerals used for the generation
of nuclear energy are uranium and thorium. Uranium is a relatively common element in the crust of
the Earth. It is a metal approximately as common as tin or zinc, and it is a constituent of most rocks
and even of the sea. Australia has a substantial part (about 31 percent) of the world's uranium,
Kazakhstan 12 percent, and Canada and Russia 9 percent each. Known uranium resources have increased
almost threefold since 1975.
• Today uranium is the only fuel supplied for nuclear reactors. However, thorium can also be utilized as a
fuel for CANDU (Canada Deuterium Uranium) reactors or in reactors specially designed for this
purpose. Neutron efficient reactors, such as CANDU, are capable of operating on a thorium fuel cycle,
once they are started using a fissile material such as U-235 or Pu-239. Then the thorium (Th-232) atom
captures a neutron in the reactor to become fissile uranium (U-233), which continues the
reaction. Thorium is about 3.5 times more common than uranium in the Earth's crust. Present
knowledge of the distribution of thorium resources is poor because of the relatively low-key exploration
efforts arising out of insignificant demand. India and Australia are believed to possess about 300,000
tonnes each; i.e. each country possessing 25% of the world's thorium reserves. Hence statement 1 is
correct.
• India has relatively modest reserves of uranium. India's uranium resources are modest, with 102,600
tonnes U (tU) as reasonably assured resources (RAR) and 37,200 tonnes as inferred resources in situ in
January 2011. However, the department of atomic energy claims to have reserves of 1, 86, 653 tU in
2013. Andhra Pradesh followed by Jharkhand and Meghalaya in that order is top state with the
largest uranium reserves. Hence statement 2 is not correct.
• Indian interest in thorium is motivated by their substantial reserves. Department of Atomic Energy has
established the presence of 10.70 million tonnes of Monazite ore, found in beach and river sand in the
country, which contains 9,63,000 tonnes of Thorium Oxide (ThO2) in 2009. India Monazite contains
about 9-10% of ThO2 and about 8,46,477 tonnes of thorium Metal can be obtained from 9,63,000 tonnes.
In 2013, total Monazite reserves are estimated to be 11.93 million tonnes.
• Thorium is mainly obtained from monazite and ilmenite in the beach sands along the coast of
Kerala and Tamil Nadu. World’s richest monazite deposits occur in Palakkad and Kollam districts of
Kerala, near Vishakhapatnam in Andhra Pradesh and Mahanadi river delta in Odisha. Hence statement 3
is correct.

Q 81.D
• Ports are classified on the basis of various parameters like the types of traffic which they handle, the cargo
handled, on the basis of location, and on the basis of specialized functions performed.
• The types of the port on the basis of specialized functions are:
o Oil ports: Deal in processing and shipping of oil eg: Tripoli in Lebanon.
o Ports of Call: These are stock replenishment ports where ships anchor for refueling, watering
and taking food items. eg. Aden, now developed as a commercial port. Hence option (d) is
correct.
o Packet Station: These are concerned with the transportation of passengers and mail across water
bodies covering short distances. E.g. Dover in England.
o Entrepot Ports: These are collection centers where the goods are brought from different countries for
export. E.g. Singapore.
o Naval Ports: They serve warships and have repair workshops for them. eg: Kochi

30 www.visionias.in ©Vision IAS

Google it:- https://upscpdf.com


https://t.me/UPSC_PDF Download From > https://upscpdf.com https://t.me/UPSC_PDF

Q 82.A
• Crude petroleum consists of hydrocarbons of liquid and gaseous states varying in chemical composition,
color, and specific gravity. It is an essential source of energy for all internal combustion engines in
automobiles, railways, and aircraft. Its numerous by-products are processed in petrochemical industries, such
as fertilizer, synthetic rubber, synthetic fiber, medicines, vaseline, lubricants, wax, soap, and cosmetics.
• Statement 1 is correct: Most of the petroleum occurrences in India are associated with anticlines and
fault traps in the rock formations of the tertiary age. In regions of folding, anticlines, or domes, it occurs
where oil is trapped in the crest of the upfold. The oil-bearing layer is a porous limestone or sandstone
through which oil may flow. The oil is prevented from rising or sinking by intervening non-porous layers.
Petroleum is also found in fault traps between porous and non-porous rocks. Gas, being lighter usually occurs
above the oil. A reservoir rock is a sub-surface volume of rock that has sufficient porosity and
permeability to permit the migration and accumulation of petroleum under adequate trap conditions.
• Statement 2 is not correct: Oil exploration and production were systematically taken up after the Oil and
Natural Gas Commission was set up in 1956. Till then, Digboi in Assam was the only oil-producing region
but the scenario changed after 1956. In recent years, new oil deposits have been found in the extreme western
and eastern parts of the country. In Assam, Digboi, Naharkatiya, and Moran are important oil-producing areas.
The major oilfields of Gujarat are Ankleshwar, Kalol, Mehsana, Nawagam, Kosamba, and Lune. Mumbai
High which lies 160 km off Mumbai was discovered in 1973 and production commenced in 1976. Oil and
natural gas have been found in exploratory wells in Krishna-Godavari and Kaveri basin on the east coast.Oil
extracted from the wells is crude oil and contains many impurities. It cannot be used directly. It needs to be
refined. There are two types of refineries in India: (a) field-based and (b) market-based. Digboi is an
example of field-based and Mathura and Barauni are examples of the market-based refinery.

31 www.visionias.in ©Vision IAS

Google it:- https://upscpdf.com


https://t.me/UPSC_PDF Download From > https://upscpdf.com https://t.me/UPSC_PDF

Q 83.D
• Statement 1 is not correct: The Cropping Intensity is calculated as (Gross Cultivated Area / Net Sown
Area). Around 51% of India's geographical area is already under cultivation as compared to 11% of the
world average. The present cropping intensity of 1.36 has registered an increase of only 25% since
independence. Further, rainfed drylands constitute 65% of the total net sown area.
• Statement 2 is not correct: Net Sown Area as a percentage of total cultivated land has increased
slightly over the years. In 1950-51, it was around 70.05 % of the total cultivable area and in 2014-15 it is
around 78.4% of the total cultivable area.
• There is an urgent need to evolve and adapt to land-saving technologies. Such technologies can be
classified under two heads – those which raise the yield of any particular crop per unit area of land and
those which increase the total output per unit area of land from all crops grown over one agricultural year
by increasing land-use intensity. The advantage of the latter kind of technology is that along with
increased output from limited land, it also increases the demand for labor significantly. For a land-scarce
but labor abundant country like India, a high cropping intensity is desirable not only for fuller
utilization of land resources but also for reducing unemployment in the rural economy.

Q 84.A
• Recently the Finance Minister announced the national rollout of a ‘One Nation, One Ration Card’ system
in all states and Union Territories by March 2021. As of now (May 2020), about 20 states have come on
board to implement the inter-state ration card portability.
• Statement 1 is not correct and statement 2 is correct:: The new system, based on a technological
solution, will identify a beneficiary through biometric authentication on electronic Point of Sale
(ePoS) devices installed at the Fair Price Shops (FPS), and enable that person to purchase the quantity
of foodgrains to which she is entitled under the NFSA. Under the National Food Security Act, 2013,
about 81 crore persons are entitled to buy subsidized foodgrain — rice at Rs 3/kg, wheat at Rs 2/kg, and
coarse grains at Re 1/kg — from their designated Fair Price Shops (FPS) of the Targeted Public
Distribution System (TPDS). This system will enable migrant workers and their family members to
access PDS benefits from any Fair Price Shop in the country.
• Statement 3 is not correct: Ration card portability is aimed at providing intra-state as well as inter-state
portability of ration cards. While the Integrated Management of Public Distribution System (IM-PDS)
portal (http://www.impds.nic.in/) provides the technological platform for the inter-state portability of
ration cards, enabling a migrant worker to buy foodgrains from any FPS across the country, the other
portal (annavitran.nic.in) hosts the data of distribution of foodgrains through E-PoS devices within a state.
• The Annavitran portal enables a migrant worker or his family to avail the benefits of PDS outside
their district but within their state. While a person can buy her share of foodgrains as per her
entitlement under the NFSA, wherever she is based, the rest of her family members can purchase
subsidised foodgrains from their ration dealer back home.

Q 85.C
• Industries using weight-losing raw materials are located in the regions where raw materials are
located. Sugar mills in India are located near sugarcane producing areas. Similarly, the locations of
the pulp industry, copper smelting, and pig iron industries are located near their raw materials. In
iron and steel industries, iron ore and coal both are weight-losing raw materials. Therefore, an optimum
location for iron and steel industries should be near raw material sources. This is why most of the iron and
steel industries are located either near coalfields (Bokaro, Durgapur, etc.) or near sources of iron ore
(Bhadravati, Bhilai, and Rourkela). Similarly, industries based on perishable raw materials are also
located close to raw material sources.
• Power provides the motive force for machines, and therefore, its supply has to be ensured before the
location of any industry. However, certain industries, like aluminum and synthetic nitrogen

32 www.visionias.in ©Vision IAS

Google it:- https://upscpdf.com


https://t.me/UPSC_PDF Download From > https://upscpdf.com https://t.me/UPSC_PDF

Google it:- https://upscpdf.com


https://t.me/UPSC_PDF Download From > https://upscpdf.com https://t.me/UPSC_PDF

Google it:- https://upscpdf.com


https://t.me/UPSC_PDF Download From > https://upscpdf.com https://t.me/UPSC_PDF

Q 93.C
• Patterns of population distribution and density help us to understand the demographic characteristics of
any area. Broadly, 90 per cent of the world population lives in about 10 per cent of its land area. The 10
most populous countries of the world contribute about 60 per cent of the world’s population. Of these 10
countries, 6 are located in Asia.
• As seen in the adjoining table, the correct order of increasing population density is Asia (144 P/Km 2),
Africa (42 P/Km2) followed by Europe (33 P/Km2). Hence option (c) is the correct answer.

35 www.visionias.in ©Vision IAS

Google it:- https://upscpdf.com


https://t.me/UPSC_PDF Download From > https://upscpdf.com https://t.me/UPSC_PDF

Q 94.D
• Methane Hydrates: Methane Hydrate is a cage-like lattice of ice inside of which are trapped molecules
of methane, the chief constituent of natural gas. It is found in sea-bed that forms at low temperatures
and high pressure.
• It is also found in onshore deposits in the permafrost of northern Canada and Russia. Heating the
deposits or lowering the pressure will release gas from the solid. Hence statement 2 is correct.
• One litre of solid hydrate releases around 165 litres of gas. Every litre of solid hydrate has almost 90-
95% of methane w/w. Hence statement 1 is correct.
• India has some of the biggest methane hydrate reserves in the world. These are tentatively estimated at
1,890 trillion cubic metres.
• An Indo-US scientific joint venture in 2006 explored four areas: the Kerala-Konkan basin, the
Krishna-Godavari basin, the Mahanadi basin and the seas off the Andaman Islands.
• The deposits in the Krishna Godavari basin turned out to be among the richest and biggest in the world.
• The Andamans yielded the thickest-ever deposits 600 metres below the seabed in volcanic ash
sediments. Hence statement 3 is correct.
Q 95.C
• Recent Context: Bihar government is setting up India’s first observatory for the mammals in
Bhagalpur district at the Vikramshila Gangetic Dolphin Sanctuary (VGDS). Hence option (c) is
correct.
• The observatory is being built on the Sultanganj-Aguwani Ghat bridge over the Ganga, it will be in the
middle of the river, where the bridge’s width will be nearly 100 feet.
• The four-storey observatory will be 40 feet high, with the bridge passing through its middle.
• The observatory building will be transparent, with glass from all sides to ensure people can watch the
dolphins.
• It will give people an incentive to visit the place and see dolphins in the sanctuary without disturbing
them.
• There are also suggestions to build another dolphin observatory at the confluence of the Ganga and
Punpun rivers near Fatuha in Patna, about 5-15 dolphins are visible at the site at all times of the year.
Q 96.A
• Option (a) is the correct answer: Recently, Pakistan signed a contract with a joint venture of a
China Power (Chinese state-run firm) and the Frontier Works Organisation (FWO-a commercial arm of
Pakistan’s military) for the construction of the Diamer-Bhasha dam.
• The Diamer-Bhasha Dam is located on the Indus River in northern Pakistan between Kohistan district
in Khyber Pakhtunkhwa and Diamer district in Gilgit Baltistan.
• India has opposed the move on the grounds that the Gilgit-Baltistan region is part of the erstwhile
state of Jammu and Kashmir that was illegally occupied by Pakistan.
• India has consistently conveyed her protest and shared concerns with both China and Pakistan on all such
projects in the Indian territories under Pakistan’s illegal occupation.
• In the past too, India has opposed projects jointly taken up by Pakistan and China in Pakistan-occupied
Kashmir (POK) as part of the China-Pakistan Economic Corridor.

Q 97.D
• Recent context: A mega online challenge, SAMADHAN has been launched to test the ability of
students to innovate.
Samadhan challenge :
• The Innovation Cell of the Ministry of Human Resources Development and All India Council for
Technical Education (AICTE) has launched it in collaboration with Forge and InnovatioCuris. Hence
statement 3 is not correct.
• Under the challenge, the participating students will search and develop measures against
Coronavirus. Hence statements 1 and 2 are not correct.
• The challenge will inspire students and faculty for novel experiments and discoveries providing a strong
baseline of experimentation.
• The newly developed measures can be made available to the government agencies, health services,
hospitals, and other services for the quick solution of such epidemics.
• The challenge will also create awareness among citizens and motivate them to face challenges, prevent
crises, and help them get a livelihood.
• The success of this program depends on the effectiveness of the ideas of participating contestants with the
ability to find solutions, technically and commercially.
36 www.visionias.in ©Vision IAS

Google it:- https://upscpdf.com


Q 98.B
• Tertiary activities include both production and exchange. The production involves the ‘provision’ of
services that are ‘consumed’. The output is indirectly measured in terms of wages and salaries. The
exchange involves trade, transport, and communication facilities that are used to overcome distance.
Tertiary activities, therefore, involve the commercial output of services rather than the production of
tangible goods. They are not directly involved in the processing of physical raw materials.
• Personal services are made available to people to facilitate their work in daily life. The workers
migrate from rural areas in search of employment and are unskilled. They are employed in domestic
services as housekeepers, cooks, and gardeners.
• Hence option (b) is the correct answer.

Q 99.C
• The National Waterway 1 or NW-1 or Ganga-Bhagirathi-Hooghly river system runs from Haldia
(Sagar) to Prayagraj across the Ganges, Bhagirathi, and Hooghly river systems. Hence, statement 1
is correct.
• The NW-1 passes through West Bengal, Jharkhand, Bihar, and Uttar Pradesh. Hence, statement 2 is
not correct.
• It is 1,620 km (1,010 mi) long, making it the longest waterway in India. It is of prime importance
amongst all the national waterways considering its locational advantages. Hence, statement 3 is correct.
• Recently, two multi-modal terminals at Varanasi (Uttar Pradesh) and Sahebganj (Jharkhand) have been set
up along this waterway.
• As per the National Waterways Act, 2016, the government has declared 106 new national waterways.
Now, there are total 111 national waterways.
• Below Map shows some of the important national waterways.

37 www.visionias.in ©Vision IAS


Q 100.C
• Recent context: The Department of Science and Technology operating under the Ministry of
Science and Technology launched the Indian National Super Model. Hence option (c) is correct.
• The Model was launched to help monitor the transmission of COVID-19 infection.
• The program will act as a common platform for experts from different fields to come together and create a
model. The model will perform rigorous tests and will practice forecast routine similar to that of the
weather forecasting communities.
• The Model will integrate mathematical and statistical forecasts. This will help in predicting the possible
impacts of the spread of the disease. A consultative committee is to work closely with the experts
involved in the model. The committee will provide guidance, aggregation, and final delivery of the
supermodel.

38 www.visionias.in ©Vision IAS

You might also like